ortho uworld step 2
A 68-year-old woman comes to the office due to pain and stiffness in her fingers. The pain has been present for 5 years, and is worst at the proximal and distal interphalangeal joints Medical history is notable for type 2 diabetes mellitus, Graves disease treated with radioiodine therapy, nonalcoholic fatty liver disease, and hypertension Vital signs are normal. BMI is 34 kg/m2. The patient is in no acute distress. The image below shows examination findings in the hands, but the remainder of her physical examination is normal. Serum rheumatoid factor is negative Which of the following is the most likely diagnosis? Q A Calcium pyrophosphate crystal deposition disease Q B. Dupuytren contracture QC. Gout Q D. Neuropathic arthropathy (Charcot joint) Q E. Osteoarthritis Q F. Seronegative rheumatoid arthritis
Q E. Osteoarthritis (picture of heberden and bouchard nodules)
A 28-year-old woman comes to the clinician with pain and numbness of both hands over the last 6 months. Cold temperatures seem to precipitate these episodes. Medical history includes exercise-induced asthma, lower back pain, and hypothyroidism. The patient takes levothyroxine daily She does not use alcohol or illicit drugs .but has a 10-pack-year smoking history. Laboratory studies show normal complete blood count and chemistry panel. TSH is 4.7 µU/ml Antinuclear antibody screen is negative. Physical examination findings are shown in the image below. in addition to smoking cessation, which of the following is the best initial therapy for this patient? QA Amlodipine 0 B. Clonidine 0 C. Increase in levothyroxine dose 0 D. Lisinopril 0 E. Metoprolol
QA Amlodipine
A 75-year-old woman comes to the emergency department due to 1 day of severe low back pain The pain started after lifting a turkey from the freezer and worsens with standing, walking, and lying on her back. She had no recent falls, lower extremity weakness, or sensory loss in the legs The patient has a history of temporal arteritis, which was diagnosed several months ago and is being treated with prednisone She does not use tobacco or alcohol. Her temperature is 36.7 C (98 F), blood pressure is 140/70 mm Hg, pulse is 80/min, and respirations are 16/min. Physical examination reveals midline tenderness with palpation or percussion of the lumbar spine Ankle jerk reflexes are absent bilaterally Knee reflexes are 2+ in both legs. Flexor plantar responses are present bilaterally Muscle strength is 5/5 in both legs. Bilateral straight leg raise to 90 degrees does not increase the pain Which of the following is the is most likely diagnosis in this patient? QA Ankylosing spondylitis 0 B. Epidural abscess 0 C. Herniated disk 0 D. Lumbosacral strain 0 E. Metastatic tumor 0 F Multiple myeloma 0 G. Spinal stenosis 0 H. Vertebral compression fracture
0 H. Vertebral compression fracture Chronic/gradual VCF • Painless • Progressive kyphosis • Loss of stature Acute VCF • Low back pain & decreased spinal mobility • Pain increasing with standing, walking, lying on back • Tenderness at affected level
A 29-year-old woman comes to the physician complaining of worsening pain ,in her right knee for the last 3 months. She tried ibuprofen, but it provided little relief. She does not use tobacco or alcohol. Her mother suffers from rheumatoid arthritis. The patient's temperature is 37.2°C (99°F). On examination, the right knee is mildly swollen and tender with decreased range of motion. X-ray shows an expansile and eccentrically placed lytic area in the epiphysis of the distal femur. Serum chemistries and complete blood count are normal. Which of the following is the most likely diagnosis? QA Baker's cyst 0 B. Giant cell tumor 0 C. Osgood-Schlatter disease 0 D. Osteitis fibrosa cystica 0 E. Osteoarthritis 0 F Osteoid osteoma
0 B. Giant cell tumor Giant cell tumor of bone is a benign and locally aggressive skeletal neoplasm that usually presents with pain, swelling, and decreased range of joint motion at the involved site. It typically presents as osteolytic lesions (with a "soap-bubble" appearance on radiographs) in the epiphyseal regions of the long bones and most commonly involves the distal femur and proximal tibia around the knee joint
A 68-year-old woman comes to the emergency department in acute distress. She complains of the sudden onset of complete vision loss in her right eye. Her vision had been blurry for the past few days and acutely worsened an hour ago. The patient also describes 2 months of right-sided headaches for which she took ibuprofen without much relief. She has no nausea or vomiting but does have malaise and fatigue. Her medical problems include diet-controlled type 2 diabetes mellitus, hypertension, and degenerative joint disease. Her temperature is 37.2 C (98 9 F), blood pressure is 146/86 mm Hg, and heart rate is 78/min. Examination shows complete loss of vision in the right eye Pupils are 4 mm bilaterally Funduscopy shows a swollen pale disc with blurred margins. The heartbeat is regular, and a bruit is heard in the right subclavicular area. Motor and sensory examination is within normal limits. Erythrocyte sedimentation rate is 85 mm/h. Which of the following is the most appropriate next step in management of this patient? QA High-dose acetazolamide 0 B. High-dose intravenous methylprednisolone 0 C. Low-dose oral prednisone 0 D. Methotrexate therapy 0 E. MRI of the brain 0 F Temporal artery biopsy
0 B. High-dose intravenous methylprednisolone
A 30-year-old man comes to the office due to low back pain and stiffness for 2 months. His pain is worse in the morning and improves with activity The patient has also had intermittent diarrhea and lower abdominal pain for the past 3 months. He tried ibuprofen, which helped his back pain but made his diarrhea worse. He has not traveled outside the United States. The patient has been sexually active with the same female partner for 2 years He has no other medical problems and does not use tobacco, alcohol, or recreational drugs Vital signs are normal. Physical examination shows limited spine flexion and tenderness in the lower back. Laboratory studies show anemia and thrombocytosis Stool cultures are negative Plain radiographs reveal sacroiliac joint inflammation. Which of the following is the most likely cause of this patient's symptoms? QA Celiac disease 0 B. Inflammatory bowel disease 0 C. Paraneoplastic syndrome 0 D. Reactive arthritis from diarrhea 0 E. Rheumatoid arthritis 0 F. Spinal osteomyelitis
0 B. Inflammatory bowel disease Inflammatory bowel disease is frequently complicated by arthritis, which occurs in up to 45% of patients and can involve axial or peripheral joints Treatment with nonsteroidal anti-inflammatory drugs may exacerbate the underlying disease
A 6-year-old boy is brought to the office for follow-up for a limp. His symptoms began 2 months ago as a dull ache in the left knee with an intermittent limp The parents brought him to an urgent care center at that time and were reassured that the laboratory evaluation and x-rays of the knees and hips were normal. The pain has not worsened, but the patient now has a persistent limp with no other joint or muscle pain. He has been well except for a brief upper respiratory infection with low-grade fever about 2 weeks ago He has a history of recurrent ear infections and had tympanostomy tubes placed 2 years ago The patient takes no daily medications, and immunizations are up to date. Temperature is 37.1 C (98.8 F), blood pressure is 95/65 mm Hg, pulse is 80/min, and respirations are 16/min. BMI is at the 60th percentile. Examination shows a well-appearing and alert child. Range of motion, particularly internal rotation and abduction, of the left hip is markedly limited. The remainder of the examination is normal. Which of the following is the most likely diagnosis in this patient? QA Bacterial arthritis 0 B. Developmental dysplasia of the hip 0 C. Hematogenous osteomyelitis 0 D. Legg-Calve-Perthes disease 0 E. Slipped capital femoral epiphysis 0 F. Transient synovitis
0 D. Legg-Calve-Perthes disease Legg-Calve-Perthes disease • Idiopathic avascular necrosis of the femur • Boys age 3-12 • Insidious hip pain, limp • Restricted hip abduction, internal rotation • Positive Trendelenburg sign • X-ray o Early stages May be normal o Later stages: Femoral head flattening, fragmentation, sclerosis • MRI: Avascular/necrotic femoral head • Non-weight bearing • Splinting, possible surgical repair
A 60-year-old man comes to the urgent care center after being awakened by severe pain in his right great toe, which is suddenly swollen and very tender to the touch. He has also had occasional headaches and "unbearable" pruritus after a hot bath over the past several weeks. The patient does not use alcohol, tobacco, or illicit drugs His temperature is 36.8 C (98 3 F), blood pressure is 140/90 mm Hg, and pulse is 90/min. Cardiopulmonary examination is norma!I. The abdomen is soft and nontender, and there is no appreciable ascites. The liver span is 10 cm at the mid clavicular line, and the spleen is palpable 2 cm below the costal margin on full inspiration His right great toe is swollen and erythematous, with severe pain on any movement. Aspiration of the affected metatarsophalangeal joint shows negatively birefringent crystals and many leukocytes but no organisms Which of the following is most likely responsible for this patient's symptoms? QA Chronic kidney disease 0 B. Hemochromatosis 0 C. Hyperparathyroidism 0 D. Inherited enzyme deficiency 0 E. Myeloproliferative disorder 0 F Pernicious anemia 0 G. PortaI hypertension
0 E. Myeloproliferative disorder
A 65-year-old woman comes to the office due to progressively worsening right knee pain The patient has had the pain for 2 years but never sought medical attention and last saw a physician 5 years ago. The pain, which is worse in the evening, was initially responsive to over-the-counter analgesics, but they gradually lost effectiveness and she stopped taking them. There is no history of trauma to the joint Medical history is unremarkable. The patient recently retired from her occupation as an elementary school janitor She does not use tobacco, alcohol, or illicit drugs Blood pressure is 160/100 mm Hg and pulse is 70/min. BMI is 34 kg/m2 Knee examination shows tenderness over the medial tibial condyle and a small joint effusion. Range of motion of the knee elicits bony crepitus Which of the following is the most likely diagnosis in this patient? QA Anserine bursitis 0 B. lliotibial band syndrome 0 C. Lateral collateral ligament injury 0 D. Medial meniscal injury 0 E. Osteoarthritis 0 F Patellofemoral pain syndrome
0 E. Osteoarthritis
A 43-year-old woman comes to the urgent care center due to a 3-day history of back pain She works in a home improvement store and was lifting a bag of potting soil when she felt a sudden "grabbing" pain in her low back. There is no radiation of the pain, no lower extremity weakness, and no associated urinary symptoms The patient's medical history is notable for polycystic ovary syndrome, for which she takes oral contraceptives She does not use tobacco, alcohol, or illicit drugs. Vital signs are normal. Physical examination shows tenderness in the right lumbar paraspinal muscles and a slight reduction in lumbar lordosis. Range of motion in the lumbar spine is normal. Gait, lower extremity strength, and deep tendon reflexes are also normal. With the patient supine on the examination table, her legs are lifted sequentially to 70 degrees with the knee held straight and the ankle dorsiflexed; this causes a feeling of tightness in the hamstring muscles but no radicular radiation of pain Which of the following is the most appropriate next step in management of this patient's symptoms? QA Hydrocodone with acetaminophen 0 B. MRI of the lumbar spine 0 C. Physical therapy 0 D. Remain off work until symptoms improve 0 E. Short course of naproxen 0 F. Spinal manipulation therapy 0 G. X-ray of the lumbar spine
0 E. Short course of naproxen Low back pain Management - if acute - NSAIDS, maintain moderate activity, and muscle relaxants, spinal manipulation, brief course of opioids In chronic- • Intermittent use of NSAIDs or acetaminophen • Exercise therapy (stretching/strengthening, aerobic) • Consider: tricyclic antidepressants, duloxetine
A 55-year-old woman comes to the office with a one-week history of pain in multiple joints She has achy pain and stiffness in both wrists and multiple metacarpophalangeal and proximal interphalangeal joints in both hands. The patient's symptoms are worst in the morning and partially improve over 10-15 minutes of normal activity There is no associated fever, chills, rash, or weight loss. She works in a day care center and does not use tobacco, alcohol, or illicit drugs Vital signs are normal. On examination, there is mild swelling with no redness or tenderness of the involved joints Laboratory studies show normal blood counts and serum chemistries. Erythrocyte sedimentation rate is 12 mm/hr. Which of the following is the most likely diagnosis in this patient? QA Acute rheumatic fever 0 B. Fibromyalgia 0 C. Polymyalgia rheumatica 0 D. Rheumatoid arthritis 0 E. Systemic lupus erythematosus 0 F Viral arthritis
0 F Viral arthritis
A 35-year-old woman comes to the physician with pain and stiffness of her wrists and hand joints for the last several months. Her morning stiffness lasts for more than an hour. She also complains of joint swelling Her past medical history is significant only for a similar episode a year ago, which resolved with over the-counter ibuprofen Joint examination shows mild redness, warmth, swelling, and tenderness in the proximal interphalangeal and metacarpophalangeal joints and wrists. X-rays show periarticular osteopenia and erosions of the proximal interphalangeal and metacarpophalangeal joints The patient began taking indomethacin, which provides good relief, but symptoms recur if she skips a dose. Which of the following is the most appropriate next step in management of this patient? QA Azathioprine 0 B. Celecoxib 0 C. Etanercept 0 D. Glucocorticoids 0 E. lnfliximab 0 F. Methotrexate 0 G. No additional treatment
0 F. Methotrexate All rheumatoid arthritis (RA) patients should receive disease-modifying antirheumatic drugs (DMARDs) as early as possible in the disease course. Methotrexate is the initial DMARD of choice in most patients with active RA. Nonsteroidal anti-inflammatory drugs or glucocorticoids should be used for initial temporary symptomatic relief while awaiting response to DMARD therapy
A 45-year-old woman comes to the office due to a 3-month history of progressive exertional dyspnea and nonproductive cough She has had no chest pain or pedal edema but reports intermittent episodes of difficulty swallowing. The patient has a history of hypertension and severe gastroesophageal reflux and has had Raynaud phenomenon for 2 years She does not use tobacco or alcohol. Blood pressure is 142/86 mm Hg and pulse is 80/min. Physical examination shows the skin over her face, hands, arms, and trunk to be thick and firm. Scattered crackles are heard on lung auscultation. Which of the following is the most likely cause of this patient's current symptoms? Q A Chemical irritant pneumonitis Q B. Interstitial lung disease Q C. Left ventricular diastolic dysfunction Q D. Obstruction of medium and small bronchioles Q E. Restricted chest wall movement
Q B. Interstitial lung disease\ Systemic sclerosis (SS} is divided into diffuse cutaneous and limited cutaneous subtypes. Although there is considerable overlap in disease manifestations between the 2 subtypes, internal organ involvement (eg, interstitial lung disease} is more common and more extensive in diffuse cutaneous SS leading to a worse prognosis
A 38-year-old man comes to the office with a 6-week history of progressive right hip pain He localizes the pain to the right groin and says that it is present at rest and worse with weight bearing. The patient has no history of trauma and has not had similar symptoms before. Medical history is notable for sarcoidosis, which has required extended courses of oral glucocorticoids. His temperature is 37.2 C (98.9 F) and blood pressure is 156/86 mm Hg On examination, the patient's face is round and there is fullness in the supraclavicular area. Forced abduction and internal rotation of the femur aggravate the pain. Muscle power is slightly decreased in the proximal thigh muscles in both legs. Reflexes are 2+ throughout, and there are no sensory deficits. Plain films of the right hip show no significant abnormalities. Which of the following is the most likely cause of this patient's hip pain? Q A Bacterial infection of the joint Q B. Degeneration of articular cartilage Q C. Disruption of bone vasculature Q D. Inflammation of the trochanteric bursa Q E. Monosodium urate synovitis
Q C. Disruption of bone vasculature
A 23-year-old man comes to the office due to right wrist pain Earlier today, while running to home ,plate during a baseball game, he landed forcefully on his outstretched right hand with the palm facing down. Since then, the patient has had right wrist pain and swelling He has had partial relief with acetaminophen and local application of ice packs, but the pain still limits his activities. The patient's medical history is unremarkable. Vital signs are normal. Physical examination shows mild swelling at the dorsum of the right wrist There is maximal tenderness proximal to the base of the first metacarpal, and the pain worsens with radial deviation of the wrist Radiographs of the wrist in multiple views reveal no fracture or dislocation. Which of the following is the most appropriate next step in management of this patient? Q A Analgesics and physical therapy Q B. Local anesthetic and corticosteroid injection Q C. Surgical exploration and tendon repair Q D. Synovial fluid aspiration and analysis Q E. Thumb spica splint and repeat radiography in 2 weeks
Q E. Thumb spica splint and repeat radiography in 2 weeks Arterial supply to the scaphoid enters at the distal pole; fracture can disrupt flow to the proximal segment, leading to avascular necrosis and nonunion. Nondisplaced fractures may not be immediately visible on x-ray. If initial x-rays are negative, CT scan or MRI is recommended, or the wrist should be immobilized and x-rays repeated after 7-10 days.
A 22-year-old soccer player comes to the emergency department with knee pain During practice earlier that day, she tried to run around another player on the right and felt a popping sensation in her right knee followed by pain and rapid swelling The patient has no other injury to the knee. She has had no significant prior knee injuries. Her past medical history is unremarkable. On physical examination, the right knee is significantly swollen and tender. Ambulation appears awkward, and the patient hesitates to bear weight on the right leg. Aspiration of the knee yields grossly bloody joint fluid. Which of the following is the most likely diagnosis? QA Anterior cruciate ligament injury 0 B. Medial collateral ligament injury 0 C. Meniscal injury 0 D. Patellofemoral pain syndrome 0 E. Popliteal cyst rupture 0 F Stress fracture of the tibial plateau
QA Anterior cruciate ligament injury acl symp: Pain: rapid onset, may be severe • A "popping" sensation at the time of injury • Significant swelling (effusion/hemarthrosis) . • Joint instability
A 42-year-old woman comes to the office due to a 1-year history of intermittent skin lesions. She has had ulcerating lesions on the lower extremities associated with inguinal adenopathy and low grade fevers. Her medical history is notable for obesity, type 2 diabetes mellitus, and hyperlipidemia Current medications include metformin and rosuvastatin. The patient also frequently takes ibuprofen for joint pain She does not use tobacco, alcohol, or illicit drugs Her temperature is 37.2 C (99 F), blood pressure is 145/90 mm Hg, pulse is 90/min, and BMI is 32 kg/m2 Cardiopulmonary examination is normal. The abdomen is soft and nontender with normal bowel sounds and a palpable spleen tip on inspiration Swelling and tenderness of the hand joints are present bilaterally Skin examination shows several healed lesions on the lower legs Laboratory results are as follows: Hemoglobin 13.2 g/dl Platelets 193,000/mm3 White blood cells 1800/mm3 Neutrophils 20% Creatinine 0.8 mg/dl Antinuclear antibody negative Which of the following is the most likely cause of this patient's condition? QA Autoimmune disorder 0 B. Drug toxicity 0 C. Lymphoproliferative disorder 0 D. Myelodysplasia 0 E. ViraI infection QF Vitamin deficiency
QA Autoimmune disorder Felty syndrome • Rheumatoid arthritis o Severe erosive joint disease & deformity o Rheumatoid nodules o Vasculitis (mononeuritis multiplex, necrotizing skin lesions) • Neutropenia (ANC <1500/µL) • Splenomegaly • Anti-CCP & RF are positive in >90% of patients Diagnosis • Markedly elevated ESR, often >85 mm/hr • Peripheral smear & bone marrow biopsy to rule out other causes of neutropenia
A 55-year-old man comes to the office due to elbow pain. For the last 3 weeks, the patient has had vague, achy pain at the left elbow that radiates to the forearm and is worse with activity and at the end of the day. He has attempted treatment with over-the-counter acetaminophen without relief. The patient works as an airport baggage handler and has been seen in the office previously for minor occupational injuries. His medical history is otherwise unremarkable. He smokes a half-pack of cigarettes daily and does not use alcohol or illicit drugs Vital signs are normal. Inspection of the left elbow shows no erythema or swelling Range of motion is normal. With the elbow held in extension, passive flexion of the wrist reproduces the patient's pain. Which of the following is the most likely diagnosis in this patient? QA Cervical radiculopathy 0 B. Lateral epicondylitis 0 C. Olecranon bursitis 0 D. Osteoarthritis of the elbow 0 E. Pseudogout 0 F. Radial tunnel syndrome 0 G. Stress fracture of the radius
0 B. Lateral epicondylitis clinical presentation • Subacute to chronic lateral elbow pain • History of repetitive or forceful wrist extension • Peak incidence age 45-54 ddx • Tenderness at epicondyle & proximal extensor muscles • Pain with resisted wrist extension or supination • Pain with passive wrist flexion Management • Modified activity & ergonomics • Inelastic counterforce brace • Nonsteroidal anti-inflammatory drugs (topical or oral) • Stretching & progressive resistance exercise • Physical therapy
A 3-year-old boy is brought to the office by his mother for follow-up after an emergency department visit for a lower extremity fracture. One week ago, the patient fell while running on the stairs and was found to have a closed tibial fracture. He was placed in a cast and has been doing well since. His history is also notable for a femur fracture shortly after learning to walk at age 13 months. The patient has speech delay and mild hearing loss. His mother has had multiple fractures since childhood and has hearing loss as well. Vital signs are normal. Height and weight are at the 5th and 20th percentiles, respectively Examination shows a young boy with no dysmorphic features. Bilateral upper and proximal lower extremities have 5/5 strength. Hypermobility of the joints in the hands is noted. The right leg is casted from below the knee to the foot, and capillary refill of the toes is <2 seconds. Based on this clinical presentation, this patient most likely has a defect in which of the following genes? QA Oystrophin 0 B. Fibrillin-1 0 C. Fibroblast growth factor receptor 3 0 D. Fragile X mental retardation 1 0 E. Type I collagen
0 E. Type I collagen Osteogenesis imperfecta Pathogenesis • >90% autosomal dominant • Type I collagen gene (COL1A1) defect • Mild to moderate o Frequent fractures o Blue sclera o Conductive hearing loss o Short to normal stature o Dentinogenesis imperfecta o Joint hypermobility • Lethal (type 11) o In utero and/or neonatal fractures o Pulmonary failure
A 29-year-old woman comes to the office due to a 6 month history of numbness and pain in her upper arms. The pain is worse on the left side and is exacerbated by lifting or other activity. Associated symptoms include fatigue, fleeting joint pains, and a 5-kg (11-lb} weight loss. The patient's medical history is unremarkable, and she does not use tobacco or illicit drugs She emigrated from Vietnam 5 years ago and works in a furniture repair shop. The patient's blood pressure is 140/90 mm Hg in the right arm and 90/55 mm Hg in the left arm, and her pulse is 78/min and regular. Conjunctiva! and oral mucosa are moist and without lesions. Cardiopulmonary examination shows clear lung fields and no heart murmurs; however, a bruit is heard in the right supraclavicular fossa, and the left radial and brachia! pulses are decreased. Abdominal examination is unremarkable. No skin lesions are present Complete blood count, electrolytes, and renal function are normal. Erythrocyte sedimentation rate is 40 mm/hr. Which of the following is the most likely diagnosis for this patient? QA Aortic dissection 0 B. Coarctation of the aorta 0 C. Giant cell arteritis 0 D. Kawasaki disease 0 E. Polymyositis 0 F. Takayasu arteritis 0 G. Thromboangiitis obliterans
0 F. Takayasu arteritis
A 72-year-old man is admitted to the hospital overnight following an elective inguinal hernia repair. There are no immediate postoperative complications, but he awakens the next morning with severe right knee pain. Medical history is notable for hypertension, chronic obstructive pulmonary disease, and mild aortic stenosis. The patient smokes half a pack of cigarettes daily and drinks alcohol on social occasions. His temperature is 38.1 C (100 5 F), blood pressure is 160/110 mm Hg, pulse is 80/min, and respirations are 16/min. Physical examination shows redness and swelling of the right knee with limited motion due to pain. Diagnostic arthrocentesis is performed, with results from synovial fluid analysis as follows White blood cells 30,000/mm3 Neutrophils 90% Polarized Few rhomboid-shaped crystals microscopy Gram stain No organisms seen Which of the following is most likely associated with this patient's current condition? A Conjunctivitis 0 B. Heberden nodes 0 C. Meniscal calcification 0 D. Rheumatoid factor 0 E. Tophi 0 F T ransient bacteremi
C. Meniscal calcification [62%] Pseudogout is an acute inflammatory arthritis caused by calcium pyrophosphate crystals It often occurs in the setting of surgery or medical illness. Pseudogout is diagnosed with synovial fluid showing rhomboid-shaped, positively birefringent crystals and radiographic evidence of chondrocalcinosis (calcified articular cartilage)
A 45-year-old woman comes to the office with knee pain She has a long history of rheumatoid arthritis affecting the joints of her hands, wrists, and knees. The patient has mild residual pain and joint stiffness on low-dose methotrexate but has otherwise done well. Over the past 2 days, the pain in her right knee has been much more severe and was associated with an episode of chills. Temperature is 38.7 C (101.6 F), blood pressure is 120/70 mm Hg, pulse is 100/min, and respirations are 18/min. Physical examination shows mild swelling in the joints of the hands and wrists. The right knee is red and more swollen than on previous examinations; active and passive range of motion at the right knee is limited due to pain. After appropriate diagnostic testing, which of the following is most likely to be indicated for this patient's current condition? QA Allopurinol 0 B. Antibiotics 0 C. Anticytokine agent 0 D. Colchicine 0 E. Glucocorticoids 0 F. High-dose methotrexate
0 B. Antibiotics
A 6-year-old girl with Down syndrome is brought to the clinic due to behavioral changes Over the past month, she has stopped running, tripped frequently while walking, and accidentally knocked toys over more often than normal. The patient was toilet-trained at age 4, but recently she began having frequent urinary accidents while playing and has started wearing a diaper again. Physical examination shows an ataxic gait. Muscle tone is decreased throughout. Bilateral ankle deep tendon reflexes are hyperreflexic When the soles of the feet are firmly stroked, the big toes move upward and the other toes fan out. Urinalysis is normal. Which of the following is the most likely cause of this patient's presentation? QA Alzheimer disease 0 B. Atlantoaxial instability 0 C. Hypothyroidism 0 D. Spinal cord infarction 0 E. Tethered cord syndrome
0 B. Atlantoaxial instability
A 16-year-old boy comes to the office in December due to right knee swelling that began earlier that day following soccer practice, although he does not recall any specific injury. The patient has mild associated pain and stiffness. ln addition, over the last several months he has experienced fatigue with occasional flu-like illnesses and variable joint pains The patient's medical history is unremarkable, and he takes no medications. He is the captain of his high sclhool soccer team and spent the summer at a soccer camp in Maine. The patient is sexually active with his girlfriend and uses condoms. Vital signs are normal. Cardiopulmonary examination is normal. The patient is able to bear weight and has a grossly normal gait. Examination of the right knee shows palpable warmth and a mild effusion but no tenderness. Range-of-motion testing shows a mild decrease in flexion in the right knee compared to the left. Other joints are normal. Plain radiographs of the knee joint reveal no bony deformity Aspiration of the knee joint yields yellow-colored, translucent fluid with a leukocyte count of 20,000/mm3 (50% neutrophils) and no organisms on Gram stain. Which of the following is the most likely cause of this patient's knee swelling? QA Autoimmune synovial inflammation 0 B. Borrelia burgdorferi infection 0 C. Disseminated gonococcal infection 0 D. Meniscus tear 0 E. Prior Chlamydia trachomatis infection 0 F. Prior streptococcal throat infection 0 G. Staphylococcal joint infection
0 B. Borrelia burgdorferi infection Lyme arthritis is the most common late manifestation of Lyme disease. It presents as an inflammatory monoarticular or asymmetric oligoarticular arthritis,. most commonly in the knee. Synovial fluid shows an inflammatory profile, but Gram stain and culture are usually negative
A 68-year-old man comes to the office with a 2-day history of right foot pain and swelling The pain started acutely when he awoke in the morning Initially, the patient had pain only on dorsiflexion of the foot, but he progressively developed pain with any movement at the ankle or weight bearing The patient has not had similar symptoms before. He is normally very active, and had spent a week bicycling in Utah 3 weeks prior to this presentation. Medical history is notable for type 2 diabetes mellitus, hypertension, and hyperlipidemia Temperature is 37 C (98 6 F), blood pressure is 140/90 mm Hg, and pulse is 90/min. Examination of the right foot shows warmth, swelling, and tenderness over the dorsal surface of the foot and around the right ankle. Both dorsiflexion and plantar flexion are restricted due to pain. The skin is intact without ulcers or breaks. Pulses are full and equal in both feet, and sensory sensations are normal. X rays reveal soft tissue swelling, small tibiotalar joint effusion, and chronic calcification of the articular cartilage There are no fractures or dislocation. Which of the following is the most likely diagnosis? QA Bacterial infection of the ankle joint 0 B. Calcium pyrophosphate arthritis 0 C. Monosodium urate arthritis 0 D. Neuropathic joint 0 E. Piantar fasciitis 0 F. Stress fracture
0 B. Calcium pyrophosphate arthritis
A 23-year-old woman comes to the physician with right foot pain that started 5 weeks ago and is sharp and localized to the forefoot. She recalls no trauma or other inciting event. The patient is an avid runner and is currently training for a long-distance race. T he pain has been worsening over the past week and prevents her from doing her daily running activities. She takes no medications. She does not use tobacco, alcohol, or illicit drugs She is not sexually active, and her last menstrual period was 8 weeks ago Her temperature is 37 C (98 6 F), blood pressure is 100/60 mm Hg, pulse is 68/min, and respirations are 12/min. Her body mass index is 15 kg/m2 Examination shows normal range of motion, no erythema or bruising, and tenderness to palpation along the second metatarsal bone on the dorsal surface of the right foot. Which of the following is the most likely diagnosis? QA Morton neuroma 0 B. Plantar fasciitis 0 C. Stress fracture 0 D. Tarsal tunnel syndrome 0 E. Tenosynovitis
0 C. Stress fracture
A 66-year-old man is admitted to the hospital for left lower quadrant abdominal pain, fever, and vomiting. CT scan of the abdomen reveals diverticulitis. The patient is started on supportive care and antibiotics.. His symptoms improve over 2 days; however, on the third day of hospitalization he reports acute pain in the right ankle, which becomes "unbearable" within a few hours. The patient had a similar pain 1 year ago that resolved with over-the-counter medication. Medical history is notable for type 2 diabetes, hyperlipidemia, and hypertension. His temperature is 37.1 C (98 8 F), blood pressure is 140/90 mm Hg, and pulse is 98/min. BMI is 36 kg/m2 Examination shows a swollen, erythematous right ankle with moderate restriction of movement due to pain; the left ankle is normal. Both ankles were normal at the time of admission to the hospital. Which of the following is the most likely diagnosis? QA Enteropathic arthritis 0 B. Gout 0 C. Neuropathic joint 0 D. Osteoarthritis 0 E. Psoriatic arthritis 0 F Rheumatoid arthritis 0 G. Septic arthritis
0 B. Gout increased risk in: Medications (eg, diuretics, low-dose aspirin) • Surgery, trauma, recent hospitalization • Volume depletion • Diet High-protein foods (meat, seafood), high-fat foods, fructose or sweetened beverages • Heavy alcohol consumption • Underlying medical conditions (eg, hypertension, obesity, chronic kidney disease, organ transplant)
A 65-year-old man comes to the clinic due to left thigh pain The pain has been present for several months with constant, moderate severity. The patient has no recent history of trauma, and there is no associated weight change or loss of appetite. Medical history is significant only for high blood pressure. He does not use tobacco, alcohol, or illicit drugs Temperature is 37.2 C (98 9 F), blood pressure is 138/88 mm Hg, pulse is 80/min, and respirations are 12/min. On examination, the lower back and hips have no apparent deformity, warmth, or erythema There is normal, pain-free range of motion at the hip. His gait is mildly antalgic Lower-extremity reflexes, motor strength, and sensation are normal, and the patient has full pedal pulses. Laboratory results are as follows: Alkaline phosphatase elevated Gamma-glutamyl transferase normal Serum calcium normal Total 25-hydroxyvitamin D normal X-ray reveals cortical thickening in the left femur. Bone scintigraphy shows increased radiotracer uptake in several spots, includingnthe left femur. This patient is at highest risk of developing which of the following complications? QA Acute renal failure 0 B. Hearing loss 0 C. Liver cirrhosis 0 D. Pulmonary hemorrhage 0 E. Renal cell carcinoma 0 F Subarachnoid hemorrhage
0 B. Hearing loss
A 5-year-old boy is brought to the office due to left hip pain The patient awoke this morning with a limp and says that his left hip hurts, especially with movement or walking He played soccer outside yesterday with no injury. Two weeks ago, he had had an upper respiratory infection that has since resolved. He has had no fever, vomiting, diarrhea, or other joint pain. His grandmother has rheumatoid arthritis treated with methotrexate. Temperature is 37.2 C (99 F), blood pressure is 100/65 mm Hg, and pulse is 92/min. Physical examination shows an interactive and talkative child who walks with a limp The left hip is slightly abducted and externally rotated with mildly decreased range of motion. The patient is able to stand a111d bear weight on the left lower extremity The remainder of the examination is normal. Bilateral hip x-ray is normal. Laboratory results are as follows: Hemoglobin 12.5 g/dL Platelets 287,000/mm3 Leukocytes 8,500/mm3 Neutrophils 60% Eosinophils 2% Lymphocytes 33% Monocytes 5% Erythrocyte sedimentation rate 15 mm/h C-reactive protein 2 mg/dL (normal: 53) Which of the following is the best next step in management of this patient? QA Blood culture 0 B. Ibuprofen and close follow-up 0 C. MRI of the left hip 0 D. Serum antinuclear antibodies 0 E. Synovial fluid aspiration of the left hip
0 B. Ibuprofen and close follow-up Transient synovitis is a common cause of hip pain in children, typically occurring in boys age 3-8. The cause is unknown but symptoms may develop days to weeks after a mild viral illness, such as in this patient Synovial inflammation leads to pain, decreased range of motion, and a limp On examination, patients are well-a1ppearing and afebrile. The affected hip may be flexed, slightly abducted, and externally rotated. This position maximizes the joint space, thereby providing pain relief. c- Legg-Calve-Perthes, or avascular necrosis of the hip, may present with mild hip/knee pain and a limp in young children. Because initial x-rays may appear normal in early disease, repeat x-rays and/or MRI are indicated for persistent or worsening symptoms However, additional imaging is unnecessary for this patient with normal x-rays and symptoms lasting <1 day. (Choice D) Antinuclear antibodies may be positive in certain subsets ofjuvenile idiopathic arthritis (eg, oligoarticular, polyarticular). Although presentation may include a limp, this condition typically occurs in female toddlers and rarely involves the hip, making this diagnosis unlikely
A 16-year-old boy is brought to the office for evaluation of left thigh pain. The pain, which is dull and aching, has been present during most evenings for the last 2 months. Over the last month, the pain has worsened and is now interrupting his sleep but improves with ibuprofen. The patient has had no trauma, fever, or pain in other joints or extiremities. He plays high school basketball and states the pain does not occur during or after practice or games He has a history of moderate, persistent asthma for which he takes an inhaled corticosteroid daily and albuterol as needed; he has required a course of oral prednisone every 2-3 years since infancy Vital signs are normal. The left lower extremity shows no redness or deformity Mild tenderness is elicited with deep palpation of the left distal thigh. All extremities demonstrate normal muscle bulk and strength with full range of motion. Gait is normal. X-ray is shown in the exhibit. Which of the following is the most likely diagnosis in this patient? QA Ewing sarcoma 0 B. Osteoid osteoma 0 C. Osteomyelitis 0 D. Osteosarcoma 0 E. Stress fracture
0 B. Osteoid osteoma small, round lucency with sclerotic margins and sometimes central ossification. NSAIDs are prescribed for symptomatic relief, and serial imaging is performed to monitor the lesion. Surgical resection is typically reserved for patients with refractory symptoms, as most lesions resolve spontaneously over several years
A 46-year-old woman suffering from systemic lupus erythematosus (SLE) develops hematuria and proteinuria She was diagnosed with SLE four years ago and has been treated with hydroxychloroquine and prednisone. A renal biopsy is performed and shows diffuse proliferative glomerulonephritis She is started on cyclophosphamide and hydroxychloroquine is stopped This new therapeutic regimen puts her at an increased risk for which of the following conditions? QA Cochlear dysfunction 0 B. Optic neuritis 0 C. Bladder carcinoma 0 D. Peripheral neuropathy 0 E. Digital vasospasm 0 F. Thyroid dysfunction 0 G. Crystal arthritis 0 H. Acute pancreatitis
0 C. Bladder carcinoma
A 33-year-old African American woman comes to the office due to skin lesions and pain in her ankles for a week. She has had no cough, sore throat, shortness of breath, abdominal pain, or bowel symptoms. She reports no recent travel or illness. The patient has no other significant medical history and takes no medications. She does not ruse tobacco, alcohol, or illicit drugs She has no history of sexually transmitted diseases and has been married to a monogamous parrtner for the past 8 years. Her mother was diagnosed with ovarian cancer at age 65. Temperature is 37.2 C (98 9 F) and blood pressure is 126/76 mm Hg. On examination, the patient has multiple tender pink to reddish nodules below the knee, as shown in the exhibit Which of the following is the most appropriate next step in management? QA Antinuclear antibodies 0 B. Biopsy of skin lesions 0 C. Chest x-ray 0 D. Colonoscopy and random biopsies 0 E. CT scan of the abdomen 0 F HIV testing 0 G. Urethral and rectal swabs
0 C. Chest x-ray Erythema nodosum (EN) is a condition of painful, red or violaceous, subcutaneous nodules. It can be a sign of a more serious disease process Even in the absence of respiratory symptoms, chest x-ray should be performed in patients with EN to assess for findings consistent with sarcoidosis.
A 64-year-old man comes to the office due to increasing pain in the right groin for the past several months. The pain increases with activity, is relieved with rest, and sometimes radiates to the upper thigh The patient has no history of trauma or falls. There is no associated fever, weight loss, or change in appetite He has a history of lumbar disk herniation but has no back pain currently. Medical history is notable for type 2 diabetes mellitus and hypertension Vital signs are normal. Height is 168 cm (5 ft 5 in) and weight is 95 kg (210 lb) BMI is 34 kg/m2 Examination shows pain on passive internal rotation of the right hip, but there is no focal tenderness on palpation of the groin and hip region Reflexes are 2+ in the lower extremities symmetrically, and there are no sensory deficits. Muscle bulk, tone, and power are normal. Dorsalis pedis and posterior tibial pulses are 2+ in both legs. Which of the following is the most likely cause of this patient's hip pain? QA Aortoiliac vascular occlusion 0 B. Cutaneous nerve compression 0 C. Degenerative joint disease 0 D. Disruption of bone vasculature 0 E. Inflammation of the trochanteric bursa 0 F. Referred pain from the lumbosacral spine
0 C. Degenerative joint disease The pain of hip osteoarthritis is typically felt in the groin, buttock, or lateral hip (trochanteric) region and can radiate to the lower thigh or knee. Patients may have mild pain and brief stiffness with prolonged rest, but the worst pain usually occurs with activity and weight bearing. Examination findings often include decreased rotational range of motion, but synovitis (ie, redness, warmth) is absent Common x-ray findings include loss ofjoint space, osteophyte formation, and subchondral sclerosis. Aortoiliac atherosclerosis can cause lower extremity pain with activity. However, the pain is typically in the entire leg (buttock, thigh, and calf) and the distal pulses would be diminished. (Choice B) Meralgia paresthetica is caused by compression of the lateral femoral cutaneous neNe at the waist. It causes burning pain and paresthesias at the lateral thigh Symptoms are unaffected by motion. (Choice D) Osteonecrosis is characterized by reduced perfusion of the femoral head and collapse of the periarticular bone. Like osteoarthritis, it can present with groin pain and reduced hip range of motion. However, most patients have nocturnal or rest pain, and almost all have a history of glucocorticoid use, sickle cell disease, or heavy alcohol use (diabetes is not a significant risk factor) (Choice E) Greater trochanteric pain syndrome (formerly trochanteric bursitis) is caused by friction of the tendons of the gluteus medius and tensor fasciae lata over the greater trochanter of the femur. Pain is localized to the lateral hip and is worsened by direct pressure. (Choice F) Impingement of the lumbar neNe roots can cause referred pain to the posterior hip, thigh, and lower leg. Hip mobility is normal, but patients may have exacerbation of pain on flexion of the hip with the knee extended (straight leg raise maneuver}
A 66-year-old asymptomatic woman comes to the office for a routine preventive examination. At her last visit a year ago, she received an influenza and pneumococcal vaccine and had a normal mammogram Four years ago, the patient had a colonoscopy that revealed only internal hemorrhoids. Her medical history is significant for osteoarthritis and hypertension. The patient also has a history of menorrhagia and uterine fibroids for which she underwent a hysterectomy with removal of the cervix 17 years ago She has a 40-pack-year smoking history and consumes wine occasionally Examination shows no abnormalities. Which of the following screening tests is most appropriate for this patient? QA Chest x-ray 0 B. Colonoscopy 0 C. Dual-energy x-ray absorptiometry 0 D. Pap test 0 E. Serum calcium and phosphorus 0 F X-ray of the pelvis and spine
0 C. Dual-energy x-ray absorptiometry Screening for osteoporosis with dual-energy x-ray absorptiometry is recommended in all women age :c:65 and for younger women who have an equivalent risk of osteoporotic fracture
A 34-year-old woman comes to the clinic with a facial rash, joint pains, and oral ulcers. She has no associated fever or other systemic symptoms and otherwise feels well. Medical history is unremarkable, and the patient's only concurrent medication is an oral contraceptive She does not smoke and consumes alcohol only on social occasions. Antinuclear and anti-double-stranded DNA antibody titers are elevated. Creatinine and complement levels are normal. The patient is diagnosed with systemic lupus erythematosus, and therapy with hydroxychloroquine is initiated. Which of the following evaluations should be done periodically while the patient is taking this medication? QA Audiometry 0 B. Echocardiogram 0 C. Eye examination 0 D. Liver function panel 0 E. Pulmonary function tests 0 F Thyroid-stimulating hormon
0 C. Eye examination Retinopathy
A 68-year-old woman is brought to the clinic by her daughter due to severe pain in her fingers. Her daughter says, "Mom has had horrible problems with her joints for a long time, but she has never tried to get help " The patient adds that her fingers have become more swollen and painful over the last few weeks. She had similar symptoms in her foot last year She was given an unknown pain pill, but it was ineffective. Medical history is notable for hypertension, hypothyroidism, and early Alzheimer dementia. Vital signs are normal. Examination findings of the hands are shown in the image below (tophi) QA Calcinosis cutis 0 B. Chondrosarcoma 0 C. Gout 0 D. Osteoarthritis 0 E. Psoriatic arthritis 0 F Rheumatoid nodules
0 C. Gout Calcinosis cutis is characterized by deposition of calcium and phosphorus in the skin. It presents with scattered whitish papules, plaques, or nodules. This patient's history of painful arthritis is more typical for gout. (Choice 8) Chondrosarcomas are bone tumors occurring in the 5th or 6th decade of life. They most commonly affect the pelvis, femur, or proximal humerus and would likely not be as multifocal and bilateral as the disease pictured. (Choice D) Severe osteoarthritis can produce Heberden and Bouchard nodes, which are hard, bony nodules over the distal and proximal interphalangeal joints, respectively This patient's tophi involving several joint spaces and soft tissue structures are more consistent with gout (Choice E) Psoriatic arthritis can present with distal interphalangeal joint arthritis, asymmetric oligoarthritis, symmetric polyarthritis, spondyloarthropathy, or aggressively destructive arthritis mutilans. Most patients have established psoriasis, and nail changes (eg, pitting} are common
A 2-year-old girl is brought to the clinic by her parents for left knee pain and fever. Her knee pain began 2 months ago Her mother notes that at the time of onset the girl and several children at day care had a sore throat and a cough, but no one else has developed joint pain The girl has pain mostly in the morning but seems to be in less pain in the afternoon. She has also had daily fevers and an erythematous macular rash over her chest around bedtime for the past month. Temperature is 36.1 C (97 F), blood pressure is 90/60 mm Hg, and pulse is 114/min. Physical examination shows swelling, erythema, and warmth of the left knee with mild tenderness on passive and active motion. Similar pain is elicited with movement of the left hip She is able to walk but has a slight limp. Laboratory results are as follows Hemoglobin 10 g/dL Platelets 520,000/mm3 Leukocytes 21,000/mm3 Erythrocyte sedimentation rate 100 mm/hr Which of the following is the most likely diagnosis in this patient? QA Acute lymphoblastic leukemia 0 B. Acute rheumatic fever 0 C. Juvenile idiopathic arthritis 0 D. Lyme disease 0 E. Osteomyelitis 0 F Septic arthritis 0 G. Transient synovitis
0 C. Juvenile idiopathic arthritis Systemic-onset juvenile idiopathic arthritis is an auto inflammatory disease in children characterized by long-standing daily fever,arthritis of ?.1 joint, and a characteristic pink macular rash. Laboratory findings include leukocytosis, thrombocytosis, elevated inflammatory markers, and anemia.
A 2-month-old girl is brought to the clinic for a well-child visit Her parents are concerned about the back of her head being flat on the right The infant usually sleeps on her back in a crib but she sometimes naps in a car seat or stroller. She is breastfeeding well and receives vitamin D supplementation Birth history is significant for a full-term cesarean delivery due to breech presentation Physical examination shows flattening of the right occiput with anterior displacement of the right ear and forehead. The patient demonstrates a preference to tilt her head to the left while rotating the chin to the right When the head is turned to the right, there is a firm mass in the inferior portion of the left sternocleidomastoid. The mass does not transilluminate. The hips are stable with no clicks or clunks. Which of the following is the most likely underlying cause of this patient's presentation? QA Congenital lymphatic malformation 0 B. Fracture of the clavicle 0 C. Postural deformity of the neck 0 D. Premature closure of a cranial suture 0 E. Tumor of the posterior fossa
0 C. Postural deformity of the neck This infant with a neck mass, ipsilateral head tilt, and contralateral chin deviation has congenital muscular torticollis (CMT). CMT is a postural deformity in which the sternocleidomastoid (SCM) muscle is tight and contracted, likely due to intrauterine crowding (eg, breech positioning [as in this patient], multiple gestation, oligohydramnios). Associated conditions that are also related to prenatal positioning include developmental dysplasia of the hip, metatarsus adductus, and clubfoot. CMT is evident from birth but typically comes to medical attention at age 1-6 months with a head preference to one side. When an infant lies down with the head facing the preferred side, positional plagiocephaly often occurs, which entails flattening of the head on that side as well as anterior displacement of the ear and forehead ipsilateral to the flattening Limited range of motion of the neck is also present on examination, and SCM thickening or a well-circumscribed mass from muscle fibrosis may be palpable. Treatment strategies include positioning (eg, increased tummy time), passive stretching, and physical therapy Missed or delayed diagnosis can lead to craniofacial asymmetry Cystic hygromas are congenital lymphatic malformations located in the posterior triangle of the neck. They are often detected prenatally and associated with aneuploidy. Postnatal examination shows a fluctuant mass that transilluminates. (Choice B) Clavicle fractures can occur from birth injury (eg, shoulder dystocia) and present with crepitus, swelling, and decreased range of motion of the ipsilateral upper extremity Clavicle fractures do not affect the neck. (Choice D) Craniosynostosis (premature closure of cranial sutures) results. in skull deformity but not anterior displacement of the ear and forehead. In addition, head tilt and a neck mass would not be seen. (Choice E) Acquired torticollis from a posterior fossa tumor is extremely rare and may present with head tilt secondary to tonsillar herniation or compensation for visual disturbances. Neurologic findings (eg, cranial nerve palsy) would be expected on examination, not a neck mass
A 67-year-old man comes to the physician due to 2 days of back pain. He was moving boxes in his garage when the pain started. The pain is not relieved by lying down and increases in intensity when straini111g and coughing. He cannot sleep during the night due to pain, and has taken several acetaminophen tablets without relief. He has never had such pain before. Physical examination shows 2+ symmetrical knee and ankle jerk reflexes. Straight leg raise is negative. He has point tenderness to palpation and percussion along the midline at the fourth lumbar vertebra. Which of the following is the most likely underlying etiology of this patient's current condition? QA Apophyseal joint arthritis 0 B. lntervertebral disk degeneration 0 C. Ligamentous sprain 0 D. Loss of bone mineral density 0 E. Nerve root demyelinization
0 D. Loss of bone mineral density his patient's presentation-acute back pain and point tenderness after strenuous activity-suggests a vertebral compression fracture(VCF). Patients with gradual onset VCF can be asymptomatic However, an acute VCF can present with sudden onset of low back pain and decreased spinal mobility after bending, coughing, or lifting. The pain typically increases with standing, walking, or lying on the back. Examination can show tenderness at the affected level. Each VCF can decrease a patient's height by .c:_1 cm and lead to kyphosis, which is associated with decreased respiratory capacity and increased risk for atelectasis pneumonia. Kyphosis can also cause a protuberant abdomen, early satiety, and weight loss. VCF is associated with a significantly increased risk for future vertebral and nonvertebral fractures. The most common cause of nontraumatic VCF is osteoporosis, which is characterized by progressive loss of bone mineral density and loss of normal bony architecture. VCF can also be due to osteomalacia, which is usually caused by inadequate levels of vitamin D, calcium, or phosphorus and results in decreased bone mineralization and increased fracture risk. Other risk factors for VCF include trauma, infection(eg, osteomyelitis), malignancy with bone metastases, and metabolic abnormalities (eg, hyperparathyroidism) (Choice A) Apophyseal joint arthritis indicates spondyloarthropathy, such as ankylosing spondylitis (AS). AS typically presents with pain and progressively limited back motion, and is more common in younger men(< age 40) In addition, the pain of AS is typically worse in the morning and improves throughout the day. (Choice B) Lumbar disk degeneration can cause low back pain in the elderly, but the pain usually worsens with activity and is relieved with rest. This patient's nighttime pain makes this less likely. (Choice C) Ligamentous back sprain usually occurs after a specific event or action. Patients usually have pain that increases with movement and decreases with rest In addition, the pain is typically in the paraspinal area without significant tenderness to palpation of the vertebra, as seen in this patient. (Choice E) Nerve root demyelination can be due to immune-mediated (eg, Guillain-Barre), toxin or medication-induced, metabolic, hereditary, or infectious processes Patients usually develop neurologic deficits that correspond to the affected nerve roots(eg, paresthesias, lower extremity weakness). In addition, nerve root demyelination does not typically cause back pain or tenderness to palpation
A 65-year-old man comes to the office due to a 4-month history of periodic back pain radiating to his buttocks and thighs The pain is exacerbated by walking or prolonged standing, although he can tolerate bicycling without significant discomfort. Associated symptoms include occasional tingling and numbness in both lower extremities. Medical history is notable for benign prostatic hyperplasia, hypertension, and hypercholesterolemia, for which he takes appropriate medications. The patient does not use tobacco, alcohol, or illicit drugs His blood pressure is 140/80 mm Hg, pulse is 76/min, and respirations are 14/min. On examination, distal pulses are full and symmetric Neurologic examination shows normal motor strength, deep tendon reflexes, and plantar reflexes in the lower extremities bilaterally. Which of the following is the most likely cause of this patient's condition? QA Cervical spondylotic myelopathy 0 B. Iliac artery atherosclerosis 0 C. Lumbar disk herniation 0 D. Lumbar spinal stenosis 0 E. Metastatic disease 0 F. Seronegative spondyloarthritis 0 G. Vitamin B,2 deficiency
0 D. Lumbar spinal stenosis lumbar spinal stenosis (SS) SS is caused by narrowing of the spinal canal, leading to compression of one or more spinal roots. It is primarily seen in degenerative arthritis with osteophyte formation affecting the facet joints (spondylosis). However, other factors may contribute, including hypertrophy of the ligamentum flavum, bulging of the intervertebral discs, and spondylolisthesis (displacement of one vertebral body relative to another) Mostpatients are over age 60 Choice A) Cervical spondylotic myelopathy presents with weakness, paresthesias, and loss of fine motor control. Neck and upper extremity symptoms are usually present, and patients will show upper motor neuron signs (eg, hyperreflexia, upgoing plantar reflex) (Choice C) Lumbar disk herniation typically causes acute back pain with unilateral radiation down the sciatic nerve to the foot (sciatica) It usually follows an inciting event, and lumbar flexion makes the pain worse. (Choice E) Vertebral metastasis presents as dull, non-radiating pain that is worse at night and not related to position or activity. Patients often have a known malignancy or systemic symptoms (eg, weight loss) (Choice F) Seronegative spondyloarthritis (eg, ankylosing spondylitis) causes pain and progressive limitation of back motion, and most often occurs in young men. Symptoms are worse with rest and improve with activity (Choice G) Vitamin B12 deficiency can cause symptoms of peripheral neuropathy but does not usually cause back pain.
A 42-year-old male presents to your office complaining of back pain that started two days ago after carrying heavy packages. He denies any weakness or sensory changes in his legs. His past medical history is insignificant He is not taking any medications and denies drug abuse. His temperature is 36. 7° C (98° F), blood pressure is 120/76 mmHg, pulse is 80/min, and respirations are 16/min. Physical examination reveals paravertebral tenderness. Lower extremity power is 5/5 and the deep tendon reflexes are 2+. Babinski's sign is negative Straight-leg raising test is negative at 90 degrees. What is the most probable diagnosis in this patient? QA Multiple myeloma 0 B. Ankylosing spondylitis 0 C. Compression fracture of the vertebrae 0 D. Lumbosacral strain 0 E. Herniated disk 0 F Metastatic tumor 0 G. Increased lumbar lordosis 0 H. Spondylolisthesis 0 I. Epidural abscess 0 J. Spinal stenosis 0 K Abdominal aortic aneurysm 0 L Cauda equina syndrome 0 M. Transverse myelitis 0 N. Vertebral body osteomyelitis
0 D. Lumbosacral strain Lumbosacral strain is the most common cause of back pain. It is estimated that the lifetime risk of lumbosacral strain is close to 80%. The clinical scenario described is typical The pain starts acutely after physical exertion, and it is concentrated in the lumbar area, usually without radiation to the thighs Physical examination reveals local tenderness and contraction of the paraspinal muscles. A straight-leg raising test and neurologic examination are typically normal. The treatment includes NSAIDs and early mobilization.
A 40-year-old woman comes to the office due to pain in her right hip for the past 4 weeks. The pain increases with weight bearing She has no morning stiffness, warmth, or erythema, and no pain in other joints. The patient has no history of significant trauma to the hip and has not had similar pain before. She was diagnosed with systemic lupus erythematosus 7 years ago Her current medications include low-dose prednisone, hydroxychloroquine, and lansoprazole. The patient does not use tobacco, alcohol, or illicit drugs She is afebrile, and vital signs are normal. There is no local tenderness over any joints, and range of motion of the right hip is normal. Plain film radiograph of the hip is normal. Which of the following is the best next step in management of this patient? QA Bone densitometry 0 B. Increasing the dose of prednisone 0 C. Joint aspiration 0 D. MRI of the hip 0 E. Naproxen and reassurance
0 D. MRI of the hip Avascular necrosis
A 22-year-old man comes to the office due to intermittent right knee pain that has been increasing for the past 8 weeks. The pain began after the patient participated in a recreational soccer tournament, but he does not recall any significant injuries He has intermittent, moderate pain associated with a sensation of "catching" in the knee while walking At times he is unable to completely extend the knee. The patient also had swelling at the joint several days after the pain began, but the swelling slowly resolved. The symptoms affect his daily routine, especially outdoor activities, which he has not done since the injury. He has taken over-the-counter acetaminophen with inconsistent relief. On examination, his gait is grossly normal. There is full, pain-free range of motion of the knee with no erythema, warmth, or swelling. No ligamentous laxity is found with varus or valgus stress, or with anterior or posterior traction on the lower leg With the knee held in internal and external rotation, flexion and extension at the knee elicits moderate pain and crepitus X-ray of the knee is normal. Which of the following is the best next step in management of this patient? QA Arthrocentesis 0 B. lntraarticular glucocorticoid injection 0 C. Joint rest and naproxen only 0 D. MRI of the knee 0 E. Radionuclide bone scan
0 D. MRI of the knee Meniscus tear
A 6-year-old boy is brought to the pediatrician with leg pain Over the last few months, he has had pain in both legs that occurs only at night. The pain is worst in his thighs although he occasionally has pain in his calves as well. The episodes last a few hours each and improve with massage and over-the-counter medications. The pain has remained unchanged over the past few months. When he has the pain, the patient does not feel an urge to move the legs He has no leg pain during the day. He is able to walk, run, and play at school without any complaints, and his mother has noticed no change in his activity level. He has no fever, chills, or weight loss and there is no history of recent trauma. On examination, the boy has full range of motion of the hips and knees bilaterally No swelling or tenderness to palpation is noted and he has a normal gait His mother is very worried because his 9-year-old cousin died of leukemia 6 months earlier. Which of the following is the most appropriate next step in management of this patient? QA Blood cultures 0 B. Bone scan 0 C. Complete blood count 0 D. Observation and reassurance 0 E. Plain radiographs 0 F Psychiatric evaluation
0 D. Observation and reassurance Growing pains are a common musculoskeletal complaint in children, occurring in approximately 10%-30% of children age 2-12 years The etiology of growing pains is unknown, but they are unrelated to growth, despite their name. The diagnosis of growing pains can be made clinically (Table) (occurs primarily at night & resolves by morning-->• Affects lower extremities (eg, thighs, calves),usually bilateral • Normal physical examination & activityin the absence of systemic symptoms and abnormal examination finding) Laboratory studies and radiographs are not necessary Management of growing pains consists of parental education and reassurance along with massage, muscle-stretching exercises, and administration of over-the-counter analgesics Children with growing pains should be followed closely to monitor for pain that increases in frequency or intensity, which may warrant further evaluation Bone scans are used in the diagnosis of metastatic bone disease or osteomyelitis, both of which typically present with unilateral pain and systemic symptoms such as fever or weight loss. Bone scans are not indicated in children suspected of having growing pains X-rays can be used to diagnose both benign (eg, osteoid osteoma) and malignant (eg, osteosarcoma) bone tumors. Although osteoid osteomas frequently present with limb pain that is worse at night and responds to treatment with nonsteroidal antiinflammatory drugs, they are most common in the second decade of life, are often associated with unilateral pain, and may cause a limp and point tenderness on examination. X-rays are indicated in children with systemic symptoms, unilateral limb pain, limp, limitation of activities, or abnormal physical examination findings This patient has none of these
A 75-year-old man comes to the office for a routine preventive examination. He has no ongoing symptoms The patient's medical history is significant for osteoarthritis, hypertension, and hyperlipidemia Current medications include hydrochlorothiazide and atorvastatin. He does not smoke and consumes 1 or 2 beers on weekends. The patient walks 2 miles every morning and eats a balanced diet Vital signs are within normal limits. His cardiopulmonary examination is normal, and his abdomen is soft and nontender. Rectal examination shows a diffusely enlarged prostate without nodules. The distal interphalangeal joints are enlarged, and his gait is normal. Laboratory results are as follows:Elevated alkaline phosphatase • Elevated bone turnover markers (eg, PINP, urine hydroxyproline) • Calcium & phosphorus normal Which of the following is the most likely cause of the elevated alkaline phosplhatase in this patient? QA Adverse effect of medication 0 B. Alcohol use 0 C. Metastatic bone disease 0 D. Paget disease of bone 0 E. Plasma cell neoplasia
0 D. Paget disease of bone
A 27-year-old man comes to the office due to pain on the medial side of the tibia just below the knee. The pain began after he fell while playing football 2 weeks ago and has been continuous since then. The patient's pain is sharp but does not radiate. He otherwise feels well, and medical history is not significant. On examination, there is localized tenderness at the upper tibia below the knee joint with no associated redness, warmth, or swelling. His gait is normal. A valgus stress test has no effect on the pain X-ray of the knee and tibia reveals no abnormalities. Which of the following is the most likely cause of this patient's symptoms? QA Medial collateral ligament sprain 0 B. Medial compartment osteoarthritis 0 C. Patellofemoral syndrome 0 D. Pes anserinus pain syndrome 0 E. Prepatellar bursitis
0 D. Pes anserinus pain syndrome This patient with medial knee pain and focal tenderness has pes anserinus pain syndrome (PAPS) This condition is often referred to as anserine bursitis, but most patients do not have true inflammation in the bursa, and multiple regional structures can contribute to the pain The pes anserinus is formed by the conjoined tendons of gracilis, sartorius, and semitendinosus. The anserine bursa is located anteromedially over the tibial plateau, just below the joint line of the knee and deep to the pes anserinus. PAPS can be caused by an abnormal gait, overuse, or trauma. Localized pain is typical over the anteromedial tibia and is often exacerbated by pressure from the opposite knee while lying on the side. Examination shows a well-defined area of tenderness over the medial tibial plateau just below the joint line. A valgus stress test does not aggravate the pain, indicating no medial collateral ligament involvement (Choice A). The diagnosis is primarily based on clinical features, although x-ray can exclude concurrent osteoarthritis of the knee
A 21-year-old soldier comes to the medical clinic due to right foot pain She is in her second month of basic training and has had pain for the last 3 weeks. The patient has no history of acute trauma. The pain initially occurred only with activity but is now present even at rest Medical history is unremarkable, and she takes no medications. Vital signs are normal. Examination shows swelling and warmth in the foot and point tenderness over the second metatarsal. Plain films of the foot show a hairline fracture of the shaft of the second metatarsal. Which of the following is the most appropriate next step un management? QA Bone scan 0 B. MRI of the foot 0 C. Plaster cast 0 D. Rest and analgesics 0 E. Surgical intervention
0 D. Rest and analgesics Stress fractue clinically: Insidious onset of localized pain • Point tenderness at fracture site • Possible negative x-ray in the first 6 weeks management Reduced weight bearing for 4-6 weeks • Referral to orthopedic surgeon for fracture at high risk for malunion (eg, anterior tibial cortex, 5th metatarsal)
A 43-year-old man comes to the physician reporting acute-onset right knee pain. He was diagnosed with diabetes mellitus a year ago and takes metformin. He does not use tobacco, alcohol, or illicit drugs The patient is in a monogamous relationship. His father also has diabetes. His temperature is 37 C (98.6 F), blood pressure is 134/86 mm Hg, pulse is 86/min, and respirations are 16/min. BMI is 26 kg/m2 Physical examination shows a slightly swollen and tender right knee and mild hepatomegaly Right knee x-ray reveals chondrocalcinosis and a moderate effusion. Appropriate analgesic is administered forjoint pain. Which of the following is the best next step in management of this patient? QA Anticitrullinated peptide antibodies 0 B. Antismooth muscle antibodies 0 C. Liver biopsy 0 D. Serum iron studies 0 E. Serum uric acid level 0 F Slit-lamp eye examination
0 D. Serum iron studies his patient has acute monoarticular arthritis with chondrocalcinosis (calcified articular cartilage on radiographs), diagnostic of calcium pyrophosphate dihydrate crystal deposition (CPPD) disease (pseudogout). Patients with pseudogout should be evaluated for secondary causes such as hyperparathyroidism, hypothyroidism, and hemoc'hromatosis. Given this patient's recently diagnosed diabetes mellitus and hepatomegaly, hereditary hemochromatosis (HH) is highly likely HH-induced iron deposition in the synovial fluid appears to promote CPPD. Diabetes in HH appears to be due primarily to loss of insulin secretion and often requires injectable insulin; however, mild or early disease is frequently managed with oral agents
A 39-year-old paleontologist complains of right-sided hip pain that makes it very difficult for him to lie on his right side while sleeping He localizes the pain to the outer surface of his thigh. He was recently diagnosed with hypertension and hyperlipidemia. He takes hydrochlorothiazide and atorvastatin. He has smoked one pack of cigarettes daily for 15 years. He does not use alcohol or illicit drugs. Which of the following is the most likely cause of his pain? QA Slipped femoral epiphysis 0 B. Paget's disease 0 C. Peripheral vascular disease 0 D. Trochanteric bursitis 0 E. Hip osteoarthritis 0 F. Femoral nerve lesion
0 D. Trochanteric bursitis Trochanteric bursitis is inflammation of the bursa surrounding the insertion of the gluteus medius onto the femur's greater trochanter. Excessive frictional forces secondary to overuse, trauma, joint crystals, or infection are responsible Patients with this condition complain of hip pain when pressure is applied {as when sleeping} and with external rotation or resisted abduction.
A 3-year-old girl is brought to the emergency department for acute onset of right elbow pain. She was playing in the snow and started crying when her father pulled her by the arm to take her inside. On examination, the girl keeps her hand in a pronated position, refuses to use her hand, and cries when her elbow is moved. There is no swelling or deformity Palpation of the radial head elicits mild tenderness. A click is felt when the forearm is hyperpronated, and the child stops crying and starts to use her arm. What is the most likely etiology of this child's symptoms? QA Lateral epicondylitis 0 B. Medial epicondylitis 0 C. Midshaft forearm fracture 0 D. Panner disease 0 E. Radial head subluxation 0 F Supracondylar fracture
0 E. Radial head subluxation
A 1-month-old girl is brought to the office for a routine newborn examination. She was born at 39 weeks gestation to a 26-year-old primigravida by cesarean section for breech presentation. The pregnancy and delivery were otherwise uncomplicated, and prenatal ultrasound revealed normal fetal anatomy The infant is formula fed every 3-4 hours and sleeps 5-6 hours overnight The patient lives at home with her parents and grandparents She received the hepatitis B vaccine at age 5 days. Physical examination shows an alert infant who is moving all extremities equally There are 4 skin creases along the patient's inner left thigh and 1 on the right thigh When supine with the knees flexed, the left knee appears to be inferior to the right knee. Skin examination shows a nonblanching, blue-grey macule with indefinite borders over the sacrum. The rest of the examination is normal. Which of the following is the most appropriate next step in management of this patient? QA MRI of lumbosacral spine 0 B. Reassurance 0 C. Repeat examination in 2 weeks 0 D. Ultrasound of hips 0 E. Ultrasound of lumbosacral spine 0 F X-ray of hip
0 D. Ultrasound of hips This patient's presentation is concerning for developmental dysplasia of the hip (DOH), an abnormal development of the femoral head and acetabulum. Risk factors include breech presentation, female sex, white ethnicity, and family history of DOH. Because many patients have no risk factors, all infants should have serial hip examinations from birth until age 1. The Barlow and Ortolani maneuvers assess joint stability, and a palpable "clunk" should prompt referral to an orthopedic surgeon These maneuvers may be negative or equivocal, and �dditional signs, such as leg-length discrepancy (eg, Galeazzi test) or asymmetric inguinal skin folds abnormal examination should undergo bilateral hip ultrasonography After age 4 months, when the femoral head and acetabulum are ossified, x-rays are preferred to evaluate acetabular development and positioning (Choice F). Confirmed DOH is treated with a Pavlik harness, a splint that holds the hip in flexion and abduction while preventing extension and adduction.
A 65-year-old man comes to the office due to right upper extremity pain after falling on his right outstretched hand 2 days ago. The patient has mild abrasions over his wrist and elbow with full range of motion and no tenderness over these joints He has severe pain when he tries to lift his arm above shoulder level or when he pulls or pushes with his right arm. He has been unable to sleep the last 2 nights due to pain and is unable to lie on the affected side. The patient has a 40-pack-year smoking history His father died of multiple myeloma Vital signs are normal. Examination shows limited active abduction and external rotation of the humerus; however, passive motion is comparable to the contralateral side. Palpation of the right shoulder, collarbone, and neck reveal no step-off deformities or point tenderness. Radial pulse, extremity sensation, muscle tone, and bulk are normal. Which of the following would most likely confirm the diagnosis in this patient? QA Bone scan 0 B. Chest x-ray 0 C. CT scan of the shoulder 0 D. MRI of the ceNical spine 0 E. MRI of the shoulder 0 F Shoulder x-ray
0 E. MRI of the shoulder This patient has post-traumatic shoulder pain with limited active abduction, most likely due to a rotator cuff tear. Rotator cuff tears are most common in patients age >40, often after a fall on an outstretched arm. Patients typically have pain at the lateral shoulder that is worsened with raising the arm overhead or with external rotation and abduction. Examination shows the drop arm sign (inability to smoothly control shoulder adduction) and weakness with external rotation (passive range of motion is normal). Rotator cuff tendinopathy (tendinitis) may also present with pain on abduction, but tendinopathy without tear does not cause weakness. X-ray of the shoulder in a rotator cuff tear is often performed as an initial study to rule out fracture; in large, chronic tears it may demonstrate migration of the humoral head, but it has low sensitivity for confirming the diagnosis of an acute tear (Choice F) MRI can visualize the soft tissues around the humeral head and can accurately diagnose a rotator cuff tear. Treatment of an acute tear usually involves surgery, with best results if performed within 6 weeks of the injury. Bone scans are most often used to diagnose infectious (eg, osteomyelitis) or metastatic disease. However, these conditions would not usually present with acute post-traumatic pain and decreased range of motion. (Choice B) With this patient's extensive smoking history, lung cancer involving the brachia! plexus (Pancoast syndrome) could cause shoulder pain However, this would typically present with radicular pain radiating to the wrist, atrophy of the muscles, and additional compressive symptoms (eg, edema). (Choice C) CT of the shoulder is effective in diagnosing fractures and other disorders involving cortical bone but is less sensitive than MRI for visualization of tendons and other soft tissue structures. (Choice D) MRI of the spine would identify the cause of cervical radiculopathy This condition usually results from disc herniation (or osteophytes) and presents with arm weakness, paresthesias, and neck pain
A 52-year-old woman comes to the physician with a rash over her face that began a few weeks ago Lately, the patient has also been having difficulty rising from a seated position and climbing stairs. On examination, an erythematous rash on the upper chest and violaceous periorbital edema are present Her vital signs are within normal limits. Examination shows symmetric proximal muscle weakness in the legs This patient's condition is most often associated with which of the following? QA Alveolar hemorrhage 0 B. Aortic aneurysms 0 C. Carpal tunnel syndrome 0 D. Glomerulonephriiis 0 E. Malignancy 0 F. Sudden visual loss
0 E. Malignancy dermatomyositis
A 30-year-old woman comes to the emergency department due to left knee pain. She was participating in a recreational hockey tournament when an opposing player struck her from the side, and the 2 players fell against the wall. The patient immediately felt a popping sensation in her knee and required assistance off the ice. She has had difficulty with ambulation since then. The patient had partial relief of pain with ice and elevation of the left leg but noticed that her knee was somewhat swollen the next morning. On examination, she has a small left knee effusion without erythema or palpable warmth. There is tenderness at the medial aspect of the knee at the joint line. With the patient standing on the left leg and the knee bent slightly, internal rotation on the knee elicits a locking sensation and significant, sharp pain. An x-ray of the knee joint shows no abnormalities. Which of the following is the most likely diagnosis? QA Anserine bursitis 0 B. Anterior cruciate ligament tear 0 C. lliotibial band syndrome 0 D. Medial collateral ligament tear 0 E. Medial meniscus tear 0 F Patellar dislocation 0 G. Patellar tendon rupture 0 H. Patellofemoral pain syndrome
0 E. Medial meniscus tear Meniscal tear thessaly test • Patient stands on 1 leg with knee flexed 20 degrees • Patient then internally & externally rotates on flexed knee McMurray test • Passive knee flexion & extension while !holding the knee in internal or external rotation Pain, clicking, or catching indicates meniscal tearT
30-year-old Caucasian female patient is seen at the rheumatology clinic. She has a 4-year history of rheumatoid arthritis. Over the past year, she has noticed an improvement in her symptoms. Examination of her joints reveals less swelling and erythema than on the previous visit Laboratory studies show: CBC Hb 10 8g/dL Ht 32% MCV 104 fl Platelet count 226,000/cmm Leukocyte count 7,500/cmm Neutrophils 65% Eosinophils 1% Lymphocytes 28% Monocytes 6% Serum Serum Na 140 mEq/L Serum K 3.9 mEq/L Chloride 100 mEq/L Bicarbonate 24 mEq/L BUN 16 mg/dl Serum Creatinine 1.1 mg/dl Calcium 9.8 mg/dl Blood Glucose 98 mg/dl Which of the following medications is this patient most likely taking? QA Hydroxychloroquine 0 B. Prednisone 0 C. Cyclosporine 0 D. Azathioprine 0 E. Methotrexate
0 E. Methotrexate Macrocyti. c anemia is a common side effect
A 51-year-old man complains of difficulty walking and mild right-sided foot pain for the past several weeks. The patient's medical history is significant for type 1 diabetes mellitus, hypertension, and hypercholesterolemia Physical examination shows a significantly deformed right foot and a mildly deformed left foot X-rays of the right foot suggest effusions in several of the tarsometatarsal joints, large osteophytes, and several extra-articular bone fragments. Which of the following is the most likely cause of this patient's complaints? QA Avascular necrosis 0 B. Bacterial infection 0 C. Bone demineralization 0 D. Hyperuricemia 0 E. Nerve damage 0 F Rheumatoid arthritis
0 E. Nerve damage This patient has neurogenic arthropathy (Charcot joint) due to diabetes. Pathogenesis involves decreased proprioception, pain, and temperature perception, which can occur due to diabetes, peripheral nerve damage, syringomyelia, spinal cord injury, vitamin B12 deficiency, or tabes dorsalis. As normal neurologic input is lost, patients unknowingly traumatize their weight-bearing joints This causes secondary degenerative joint disease, joint deformation, and functional limitation. X-rays will reveal loss of cartilage, osteophyte development, and loose bodies. In diabetes, vasculopathy also contributes to disease pathogenesis. Associated pain is typically mild. Management of a Charcot joint involves treating the underlying disease and providing mechanical devices (eg, special shoes) to assist in weight bearing and decrease further trauma Gout results from uric acid deposition within joints It is exquisitely painful and generally affects the great toes, ankles, wrists, and elbows. X-rays of joints affected by chronic gout reveal punched-out erosions with an overhanging rim of cortical bone known as a "rat bite" lesion Rheumatoid arthritis is a common cause of polyarticular inflammatory arthritis. It typically affects the hands, wrists, feet, and ankles, generally in a symmetric fashion. X-rays of patients with long-standing, poorly controlled disease will reveal periarticular osteoporosis, joint erosions, and joint space narrowing
An 11-month-old African-American boy is brought to the pediatrician for a well-child visit. He was born full-term and is primarily breastfed. He eats homemade baby food but not store-bought food. The infant has been growing and gaining weight well. He has started cruising but is not yet walking. He is able to say "mama" and "dada." The boy has no chronic medical issues and takes no medications. He has a cousin with achondroplasia On examination, the anterior fontanel is wide open Palpation shows pliable skull bones without step-offs Bony prominences of the costochondraljunctions are noted bilaterally Genu varum is present The remainder of his examination is normal. Which of the following is the most likely cause of this infant's findings? QA Abnormal thyroid function 0 B. Congenital infection 0 C. Nonaccidental trauma 0 D. Normal variant 0 E. Nutritional deficiency 0 F Skeletal dysplasia
0 E. Nutritional deficiency Vitamin D deficiency rickets Risk factors • • Increased skin pigmentation Exclusive breastfeeding • Inadequate sun exposure • Maternal vitamin D deficiency • Craniotabes ("ping-pong ball" skull) • Delayed fontanel closure Clinical • Enlarged • Skull (frontal bossing) • Costochondral joints ("rachitic rosary'') • Long-bone joints (wrist widening) • Genu varum • Osteopenia X-ray features • Metaphyseal cupping & fraying • Epiphyseal widening Serum • Calcium: Normal to low • Phosphorous: Normal to Ilow • Alkaline phosphatase: increased Parathyroid hormone: increased • 25-0H vitamin D: decreased Congenital hypothyroidism can present with a large anterior fontanel, lethargy, feeding difficulties, and macroglossia; it does not cause bony abnormalities. (Choice B) Congenital syphilis can present with bony abnormalities such as frontal bossing, anterior bowing of the shins ("saber shins"}, saddle-nose deformity, and notched, gap teeth (Hutchinson teeth} l\lone are seen in this patient (Choice C) Nonaccidental trauma or child abuse should be suspected in children with injuries inconsistent with their developmental stage or when the history is not consistent with the injury. Characteristic radiographic injuries include "bucket-handle" fractures, also known as classic metaphyseal lesions, and rib fractures. (Choice D) Although anterior fontanels vary in size, costochondral joint enlargement is always pathologic and should prompt evaluation for rickets. (Choice F) Skeletal dysplasias such as achondroplasia often present with bony abnormalities, including frontal bossing, limb shortening, and other dysmorphologies; none are present in this patient
A 24-year-old woman comes to the office after 6 months of burning pain in her right hand. The pain is mild and intermittent but worse at night, and is felt mainly over the first, second, and third digits of the hand. Shaking the hands or letting them hang down briefly relieves the pain There is no associated neck or forearm pain The patient attempted treatment with over-the-counter analgesics and a topical "arthritis cream" without relief. Her medical history is notable for end-stage renal disease, for which the patient underwent a living-donor transplant 2 years ago She does not use tobacco, alcohol, or illi:cit drugs. Vital signs are normal. On examination, there is no thenar or hypothenar atrophy, and thumb opposition is normal. Percussion over the volar aspect of the wrist reproduces the pain Which of the following is the most appropriate next step in management of this patient? QA Decompression surgery 0 B. lndomethacin 0 C. Local glucocorticoid injection 0 D. Oral glucocorticoids 0 E. Wrist splinting
0 E. Wrist splinting Carpal tunnel syndrome is caused by compression of the median nerve where it passes under the transverse carpal ligament in the wrist Most patients respond to nocturnal wrist splinting, but those with significant weakness or refractory symptoms may require surgical decompressio
A 54-year-old retired schoolteacher comes to the physician due to worsening low back pain. The pain started 3 weeks ago. It is continuous, does not improve with rest, and is worse at night. The patient has had little relief with over-the-counter nonsteroidal analgesics. He has no other symptoms He had a surgical resection of a lung tumor 1 year ago for non-small cell carcinoma of the lung At that time, preoperative positron emission tomography (PET) scanning did not reveal any evidence of metastasis. His vital signs are within normal limits. Examination shows local spinal tenderness at the L4-L5 level. What is the most likely cause of this patient's back pain? QA Ankylosing spondylitis 0 B. Central spinal canal stenosis 0 C. Disc herniation 0 D. Iliac artery thrombosis 0 E. Lumbar strain 0 F Metastatic disease 0 G. Osteoporotic compression fracture 0 H. Vertebral osteomyelitis
0 F Metastatic disease
A 52-year-old woman comes to the clinic with gradual-onset weakness of the leg muscles. For several months, she has had increasing difficulty climbing stairs and rising from a chair. The patient initially attributed her symptoms to being "out of shape." She has no problem chewing food, changes in bowel or bladder function, morning stiffness, or joint pain. Medications include lisinopril and amlodipine for hypertension She does not use tobacco, alcohol, or illicit drugs. Vital signs are normal. Examination shows 4/5 strength in the thigh muscles of both legs, and distal muscle strength is 5/5. Repeated muscle contractions do not alter the weakness. Mild tenderness is noted in the proximal thigh muscles. Deep tendon reflexes and sensory examination are normal, and Babinski sign is absent. Which of the following would establish a diagnosis in this patient? QA Acetylcholine receptor autoantibody test 0 B. Discontinuation of lisinopril 0 C. Electromyogram 0 D. Erythrocyte sedimentation rate 0 E. MRI of the spine 0 F Muscle biopsy 0 G. Serum electrolyte assay
0 F Muscle biopsy\ polymyositis
A 63-year-old man comes to the emergency department due to a 2-day history of right knee pain and swelling. The pain is severe enough to interfere with walking The patient does not recall any significant trnuma to the knee. Medical history is notable for type 2 diabetes mellitus, hypertension, hypercholesterolemia, and mild chronic obstructive pulmonary disease. The patient has a 40 packyear smoking history and drinks 3-5 alcoholic beverages a night. He is an industrial concrete finisher and frequently works extended hours when on a job. Temperature is 37.2 C (99 F), blood pressure is 115/75 mm Hg, and pulse is 92/min. On examination, there is mild swelling anterior to the patella, along with faint erythema and sharp tenderness. Passive range of motion at the knee is normal. Which of the following is the most likely cause of this patient's acute symptoms? QA Crystal-induced arthropathy 0 B. Fracture of the patella 0 C. Infectious arthritis 0 D. Patellar tendinitis 0 E. Patellofemoral pain syndrome 0 F Prepatellar bursitis
0 F Prepatellar bursitis This patient with acute pain and localized tenderness has prepatellar bursiti's, sometimes called "housemaid's knee." Prepatellar bursitis is common in occupations requiring repetitive kneeling, such as concrete work, carpet laying, and plumbing. While bursitis in other locations is generally noninfectious, acute prepatellar bursitis is very commonly due to Staphylococcus aureus, which can infect the bursa via penetrating trauma, repetitive friction, or extension from local cellulitis. The diagnosis should be confirmed with aspiration of bursa! fluid for cell count and Gram stain. If Gram stain and culture are negative, patients may be managed with activity modification and nonsteroidal anti-inflammatory drugs. Otherwise, patients are treated with drainage and systemic antibiotics.
A 52-year-old man comes to the physician with a long history of joint pain Tlhe patient has pain and stiffness of the small joints of his hands. The symptoms are worse in the morning and can last several hours. He also has digit swelling The patient's hands are shown in the image below:Which of the following is the most likely diagnosis? A Crystalline arthritis 0 B. Oermatomyositis 0 C. Enteropathic arthritis 0 D. Neuropathic arthropathy 0 E. Osteoarthritis 0 F Psoriatic arthritis QRheumatoid arthritis QH Sarcoidosis
0 F Psoriatic arthritis
A 9-year-old boy is brought to the physician with fever, hives, and joint pain. The pruritic rash began 2 days ago and has spread to cover his trunk and arms. Today, the boy has been having pain in the wrists and ankles. He is currently taking oral penicillin for streptococcal pharyngitis that was diagnosed 9 days ago His temperature is 38.8 C (101.8 F) Physical examination shows an ill appearing boy with a diffuse rash on the arms and back. There are palpable lymph nodes in the cervical, axillary, and inguinal regions The wrists and ankles are tender to palpation but are not swollen or erythematous. The remainder of the physical examination is normal. Which of the following is the most likely diagnosis? QA Acute rheumatic fever 0 B. Anaphylaxis 0 C. Henoch-Schonlein purpura 0 D. Infectious mononucleosis 0 E. Scarlet fever 0 F Serum sickness-like reaction 0 G. Stevens-Johnson syndrome
0 F Serum sickness-like reaction This child's symptoms (fever, urticaria, and joint pain), which began after 1 week of penicillin therapy, are concerning for a serum sickness-like reaction. A serum sickness-like reaction is a type Ill hypersensitivity reaction that occurs 1-2 weeks after administration of 13-lactams (eg, penicillin, amoxicillin, cefaclor) or trimethoprim-sulfamethoxazole. Classic manifestations include fever, urticaria, and polyarthralgia with no mucosa! involvement The patient typically appears ill
A 45-year-old man comes to the office due to a 2-month history of painless, nonpruritic, purple lesions on his legs He has also had progressive fatigue, weakness, and fleeting joint pains in his knees and elbows for the past several months. The patient has had no fever, weight loss, diarrhea, or abdominal pain Temperature is 36.7 C (98 F), blood pressure is 147/91 mm Hg, and pulse is 84/min. Cardiac examination reveals no murmurs or additional sounds. There are multiple purpuric, palpable lesions on both lower extremities that do not blanch with pressure. Laboratory results are as follows Urinalysis shows 3+ blood, 3+ protein, and few dysmorphic red blood cells. Which of the following would be most helpful in establishing this patient's diagnosis? QA ADAMTS13 activity 0 B. Anticardiolipin antibody testing 0 C. Bone marrow biopsy 0 D. Serum protein electrophoresis 0 E. Urine toxicology screen 0 F Viral hepatitis serology
0 F Viral hepatitis serology This patient's presentation is concerning for mixed cryoglobulinemia syndrome (MCS), which is caused by immune complex deposition in small- to medium-size blood vessels, leading to endothelial injury and end-organ damage. It commonly presents with fatigue; nonblanching, palpable purpura; arthralgias; renal disease (eg, hematuria, proteinuria, glomerulonephritis); and peripheral neuropathies Renal involvement is variable, but the most common manifestation is hypertension Liver involvement (eg, elevated transaminases) is common. Patients rarely may have central nervous system or pulmonary involvement MCS is most commonly associated with chronic inflammatory conditions such as hepatitis C virus (HCV) infection and systemic lupus erythematosus. Consequently, every patient suspected of having MCS should be tested for HCV, hepatitis B virus, and HIV. HGV-associated MCS immune complexes are formed from HCV, anti-HGV lgG, lgM anti-lgG antibodies (rheumatoid factor), and complement. Diagnosis of MCS can be confirmed serologically (serum cryoglobulins, low complement levels) or with a skin/renal biopsy. Treatment involves addressing the underlying disease and can also include plasmapheresis and immunosuppression (eg, glucocorticoids, rituximab) for patients with rapidly progressive or life-threatening courses
A 42-year-old man comes to the emergency department due to severe back pain following a fall. He tripped while getting out of his car and landed on his back. The patient has a 20-year history of ankylosing spondylitis, for which he takes prescribed medication regularly, but he has never had pain similar to this. His medical history is also notable for a duodenal ulcer 7 years ago The patient does not use alcohol, tobacco, or illicit drugs Vital signs are normal except for a mild, regular tachycardia. He appears to be in pain, with grimacing and diaphoresis. Examination shows midline tenderness over the upper lumbar region Spinal range of motion is restricted. There is no tenderness over the sacroiliac joints Cardiac auscultation shows an early diastolic murmur. Which of the following is the most likely cause of his current pain? QA Disc herniation 0 B. Referred visceral pain 0 C. Spinal metastasis 0 D. Spondylitis flare-up 0 E. Spondylolisthesis 0 F. Vertebral fracture
0 F. Vertebral fracture Patients with long-standing ankylosing spondylitis can develop bone loss due to increased osteoclast activity in the setting of chronic inflammation. In addition, spinal rigidity in these patients can increase the rislk of vertebral fracture, which often results from minimal trauma.
A 23-year-old woman comes to the office due to progressive low back pain and stiffness. The pain has been present for the last few years and has worsened somewhat in recent weeks. The patient has relief of the pain with physical activity and over-the-counter ibuprofen. Associated symptoms include morning stiffness that usually lasts an hour, but she has had no rash, ocular symptoms, urinary symptoms, or diarrhea. Medical history is notable only for an appendectomy at age 11. The patient has taken birth control pills for the past 2 years and is sexually active with one partner. On examination, she is afebrile with normal height and weight There is reduced forward flexion of the lumbar spine and tenderness over the lumbosacral area. The remainder of the physical examination is normal. Which of the following would be most likely to establish a diagnosis in this patient? QA Antinuclear antibodies 0 B. Erythrocyte sedimentation rate 0 C. HLA-B27 testing 0 D. Nuclear medicine bone scan 0 E. Prednisone therapeutic trial 0 F. Rheumatoid factor 0 G. X-ray of the sacroiliac joints
0 G. X-ray of the sacroiliac joints ankylosing spondylitis plain x-rays showing sacroiliitis can confirm the diagnosis
A 12-year-old male is brought to your office by his mother because of a several month history of back pain He also experienced bed wetting recently His past medical history is insignificant. He tried acetaminophen for pain relief. He does not smoke or consume alcohol. His temperature is 36.7° C (98° F), blood pressure is 110/65 mmHg, pulse is 80/min, and respirations are 16/min. Physical examination reveals a palpable "step-off' at the lumbosacral area. Straight-leg raising test is negative on both sides. Perianal sensation tested by pinprick test is decreased, but anal reflex is normal. What is the most probable diagnosis in this patient? QA Multiple myeloma 0 B. Ankylosing spondylitis 0 C. Compression fracture of the vertebrae 0 D. Lumbosacral strain 0 E. Herniated disk 0 F Metastatic or primary tumor 0 G. Increased lumbar lordosis 0 H. Spondylolisthesis 0 I. Epidural abscess 0 J. Osteoporosis
0 H. Spondylolisthesis Spondylolisthesis is a developmental disorder characterized by a forward slip of vertebrae (usually L5 over S1 ). Such a displacement results in chronic back pain and in neurologic dysfunction, if significant The area of vertebral displacement (a palpable "step-off') is detected on physical examination if the problem is severe_Other causes of back pain should be considered in this patient Mechanical causes like lumbosacral strain (Choice D) and herniateddisk (Choice E), also very common in adults, are uncommon in children_ Ankylosing spondylitis (Choice B) has slow onset, but neurologic dysfunction argues against this condition. Epidural abscess (Choice I) is usually accompanied by fever, and the source of infection is frequently present Metastatic or primary tumor (Choice F) is a possibility in this patient, but slow development of the pain and a palpable "step-off' favors spondylolisthesis as the most probable diagnosis
A 24-year-old woman comes to the office with a 4-week history of joint pain. She has had moderate, achy pain and swelling in multiple metacarpophalangeal joints of both hands, with lesser pain in the wrists, knees, and elbows. In addition, the patient experienced intermittent fevers at the onset of her pain, but has been afebrile since then. She has had relief of her symptoms with over-the-counter ibuprofen, but in the week since she scheduled her appointment, she has not required medication as the pain has resolved. The patient is married and has 2 children. Vital signs are normal. Examination shows normal range of motion in all joints tested, with no redness, warmth, or swelling in any hand joints X-rays of the hands are normal. Which of the following is the most likely diagnosis? QA Crystalline arthritis 0 B. Fibromyalgia 0 C. Gonococcal arthritis 0 D. Lyme disease 0 E. Polymyalgia rheumatica 0 F Rheumatoid arthritis 0 G. Spondyloarthropathy 0 H. Systemic lupus erythematosus 0 I. ViraI arthritis
0 I. ViraI arthritis This patient has symmetric polyarticular arthritis with a brief, self-limited course. In light of her exposure to young children, this is likely viral arthritis due to parvovirus 819. Viral arthritis is often associated with joint swelling and tenderness, but objective findings may be subtle or absent. Whereas children with parvovirus B19 typically present with the characteristic "slapped cheek" rash, adults are more likely to develop a nonspecific morbilliform exanthem and may have no skin manifestations at all. A similar arthritis may also be seen with HIV, mumps, rubella, and other viruses. The arthritis of parvovirus B19 may resemble early rheumatoid arthritis (RA) or systemic lupus erythematosus (SLE), and may be associated with-a weaklv oositive rheumatoid factor or antinuclear antibodv titer. However. RA and SLE are sianificantlv less1 associated with a weakly positive rheumatoid factor or antinuclear antibody titer. However, RA and SLE are significantly less common, usually produce more obvious synovitis, and typically follow a more chronic, protracted course (Choices F and H). In addition, SLE is usually associated with manifestations in multiple systems (eg, malar rash, oral ulcers, serositis) and is less likely to resolve spontaneously. If there is doubt regarding the diagnosis, parvovirus 819 infection can be confirmed with assay for antiparvovirus lgM or nucleic acid amplification testing
A 55-year-old woman comes to the office with right shoulder pain for the past 2 months. The patient is unable to raise her arm or reach for objects Her right hand is dominant, but she has recently been using her left more due to the problems on the right The patient lives with her disabled spouse, who had a stroke 8 months ago Before the pain, she had taken care of his daily activities and helped transfer him from his bed to wheelchair but is now unable to do so. The patient's other medical conditions include type 2 diabetes mellitus, Graves disease, hypertension, and hypercholesterolemia. On examination, there is markedly decreased passive and active abduction, flexion, and rotation at the right shoulder. Palpation of the shoulder and acromioclavicular joint do not reproduce the pain. The remainder of the physical examination is unremarkable. X-ray of the shoulder is shown below Which of the following is the most likely cause of this patient's current symptoms? QA Adhesive capsulitis 0 B. Calcific tendinopathy 0 C. Osteoarthritis 0 D. Rotator cuff tear 0 E. Rotator cuff tendonitis
Adhesive capsulitis (frozen shoulder) Adhesive capsulitis is characterized by pain and loss of distensibility of the glenohumeral joint capsule due to chronic inflammation, fibrosis, and contracture. It is often seen in patients with diabetes mellitus, th1yroid disorders, chronic immobility, rotator cuff tendinopathy, and fracture of the proximal humerus. Examination shows decreased active and passive range of motion in multiple planes X-ray findings are norm
A 65-year-old man comes to the office due to intermittent right knee pain for the last 3 or 4 months. His symptoms are worse at the end of the day, and he has had mild swelling at the knee following strenuous exertion. There is no associated fever, malaise, or rash. The patient takes metformin for type 2 diabetes mellitus. BMI is 36 kg/m2. On examination, the right knee has bony enlargement and a small effusion but no warmth or erythema Range of motion elicits pain Arthrocentesis is performed, and synovial fluid analysis shows a leukocyte count of 1800/mm3 with no crystals. Serum creatinine is 0.8 mg/dl and serum uric acid level is 5.6 mg/dl In addition to a supervised exercise program, which of the following medications is the most appropriate for relief of this patient's symptoms? QA Acetaminophen with oxycodone 0 B. Colchicine 0 C. Diclofenac 0 D. Febuxostat 0 E. Prednisone
Diclofenac first is weight loss, exercise, then nsaids, then topical agents, interarticular steroids and hyaluranic acid.
A 17-year-old female high school student comes to the office for evaluation of knee pain She has a progressive 3-month history of poorly localized, achy pain at the anterior right knee that is worse with running, sitting for an extended period, and going up or down stairs. The patient has been training intensively for competitive dance and has had to curtail her practice sessions due to a sensation that the knee is "giving away" or buckling. She initially had partial relief from over-the-counter nonsteroidal anti-inflammatory drugs, but their effectiveness has decreased. Her medical history is unremarkable, and she does not take any prescription medications. Examination shows normal gait and no visible deformity at the knees. There is mild crepitus with range of motion at the right knee. With the knee extended, compressing the patella into the trochlear groove reproduces the pain. Which of the following is the most appropriate next step in management of this patient's symptoms? A. Hinged knee brace B. lntraarticular glucocorticoid injectiOn C. Knee immobilizer 0. Oral glucocorticoid E. Patellar arthroplasty F Quadriceps strengthening exercises
F Quadriceps strengthening exercises [58%] Patellofemoral pain syndrome is a common cause of anterior knee pain in young women. It is usually due to chronic overuse or malalignment. The patellofemoral compression test can reproduce the pain Initial management includes activity modification, nonsteroidal anti-inflammatory drugs, and stretching and strengthening exercises.
A healthy 62-year-old woman comes to the physician with pain behind her right heel for a day. The pain is worse with activity and partially relieved with rest. She has no fever, back pain, swelling, or skin rash. She eats a healthy diet and walks 2 miles every morning. The patient has no chronic medical problems but was diagnosed with a urinary tract infection 4 days ago and has been taking ciprofloxacin. Her 55-year-old brother has gout Her vital signs are within normal limits. Examination shows tenderness 3 cm above the posterior calcaneus. There is no leg swelling Which of the following is the most likely cause of this patient's leg pain? QA Achilles tendinopathy 0 B. Crystal-induced synovitis 0 C. Deep venous thrombosis 0 D. Osteonecrosis 0 E. Septic arthritis
Fluoroquinolone use is associated with tendinopathy and tendon rupture (Achilles most common). Patients can develop tendon pain within 24 hours with a median of 8 days after starting the drug. Patients should stop the drug at onset of symptoms, avoid exercise and/or use of the affected area, and seek medical care for symptom evaluation and changing to a non-fluoroquinolone antibiotic
A 23-year-old man comes to the urgent care center due to a knee injury 2 days ago. He was playing basketball when he was hit on the left side by an opposing player while running The patient attempted to continue playing, but his left knee felt tight and he had to sit out the rest of the game. Over the last 2 days, the pain has been persiste111t and only partially relieved with ice and ibuprofen. He also has been using a knee brace for improved stability Examination shows mild soft-tissue swelling of the left knee without a significant joint effusion. There is tenderness at the medial joint line but no locking or catching Range of motion of the knee is near normal. Abduction (valgus) stress test shows some left knee laxity but is limited by pain. X-ray of the knee is normal. Which of the following is the most likely diagnosis in this patient? QA Anterior cruciate ligament injury 0 B. Lateral collateral ligament injury 0 C. Medial collateral ligament injury 0 D. Medial meniscus tear 0 E. Patellar tendinitis 0 F. Tibial plateau fracture
Medial collateral ligament (MCL) tear is a common knee injury caused by severe valgus stress (eg, blow to the lateral knee) or twisting injury. Examination findings may include local swelling, ecchymosis, and joint line tenderness at the medial knee. Appreciable laxity when the leg is forced into abduction (valgus stress test) is helpful for diagnosis but may be masked by swelling and muscle spasm. MRI is the most sensitive test for diagnosis but is generally reserved for patients being considered for surgical intervention. Patients with uncomplicated MCL tears can be managed nonoperatively with rest, ice, compression, and elevation (RICE measures) and analgesics with progressive return to activity as tolerated. (Choice A) Injury to the anterior cruciate ligament will produce acute, often dramatic hemarthrosis with an effusion on physical examination. The tibia will show anterior rather than valgus laxity (Choice 8) Injuries to the lateral collateral ligament are uncommon but are occasionally seen following high-velocity trauma. The knee will have varus laxity. (Choice D) The medial meniscus and MCL are intimately associated and may be injured together. A meniscus injury may show joint line tenderness, but there is typically also a small joint effusion and crepitus, locking, or catching with range of motion. Meniscal tears often occur when patients pivot on a flexed knee while the foot is planted (Choice E) Patellar tendonitis ("jumper's knee") is a chronic overuse injury characterized by anterior knee pain and tenderness. Stress testing of the ligaments is normal. (Choice F) Tibial plateau fractures can occur due to contact sports, falls, or motor vehicle accidents. However, most patients with such fractures are unable to bear weight on the knee
6-day-old boy is brought to the office for a routine visit after discharge from the hospital. He is exclusively breastfed and is gaining weight appropriately He was born at 40 weeks gestation to a primigravida who had an uncomplicated vaginal delivery Physical examination at rest shows bilateral medial deviation of the forefoot. Ankle movements appear normal, and passive and active movement of the foot results in lateral deviation of the forefoot. The remainder of the physical examination is normal. Which of the following is the best next step in management of this patient? QA Foot radiographs 0 B. Karyotype analysis 0 C. Reassurance 0 D. Serial manipulation and casting 0 E. Ultrasound of hips
Metatarsus adductus: Flexible positioning Medial deviation of forefoot Neutral position of hindfoot Reassure Congenital clubfoot: Rigid positioning Medial/upward deviation of forefoot forefoot & hindfoot Hyper-plantar flexion of foot Serial manipulation & casting; surgery for refractory cases Metatarsus adductus (MA), the most common congenital foot deformity, is characterized by medial deviation of the forefoot with a normal neutral position of the hindfoot. This deformity is usually bilateral and occurs most frequently in first-born infants, likely due to the crowded positioning in a smaller, primigravid uterus. Over 90% of cases are characterized by flexible feet that overcorrect both passively and actively into lateral deviation (abduction), as in this patient This most common form of MA corrects spontaneously; therefore, treatment is usually unnecessary Karyotyping should be considered in patients with congenital clulbfoot due to an increased risk of chromosomal anomalies. However, MA is not associated with an underlying syndrome (Choice D) Treatment of congenital clubfoot requires orthopedic evaluation as well as serial manipulation and casting soon after birth. For the minority of cases of MA that do not passively correct to a neutral position, stretching exercises and/or serial casting can be considered. (Choice E) Developmental dysplasia of the hip is confirmed by ultrasound after physical examination findings show hip instability. Developmental dysplasia is associated with breech presentation, a contraindication to vaginal delivery, and not with MA.
A 54-year-old woman comes to the office due to left wrist pain 3 days after a fall. She was walking her dog when it suddenly ran after another dog, causing her to fall. She landed forcefully on her left palm She 1had no other injuries The patient has been taking acetaminophen, which provides partial relief. She does not use tobacco or illicit drugs and drinks alcohol socially Blood pressure is 120/70 mm Hg and pulse is 84/min. Physical examination shows left wrist tenderness with maximal tenderness at the dorsoradial wrist lateral to the tendon of the extensor pollicis longus Left wrist radiographs reveal a radiolucent line across the waist of the scaphoid bone. She is treated nonsurgically with analgesics and immobilization. This patient should be monitored closely during treatment due to which of the following potential complications? QA Bacterial tenosynovitis 0 B. Fat embolism 0 C. Ganglion cyst 0 D. Nerve injury 0 E. Osteonecrosis
Osteonecrosis Bacterial tenosynovitis typically occurs following penetrating injury to the hand (eg, cat bite) or from hematogenous spread of a distant infection (eg, Neisseria gonorrhoeae) (Choice 8) Fat embolism is most common following major fracture of long bones (eg, femur) or extensive soft-tissue injury (eg,burns) (Choice C) Synovial cysts of the wrist (ganglion cysts) are most common in individuals age 15-40 and are thought to arise in most cases due to repetitive stress or inflammation. (Choice D) Lunate dislocation following a fall on an outstretched hand can cause compressive neuropathy of the median nerve. Nerve injury is not common following scaphoid fracture.
A 7-year-old boy is brought to the office for evaluation of chronic left thigh pain and a limp The pain began 8 months ago as an intermittent ache, especially after soccer practice. The patient's family thought his symptoms were growing pains and have been giving him ibuprofen as needed for pain, with some improvement Over the past month, however, the pain has been constant, and the patient recently developed a limp. He has had no fever, additional joint pain, or chronic medical problems and takes no daily medications. While walking to the examination table, the patient avoids bearing full weight on the left leg. Examination shows significantly limited range of motion of the left hip and atrophy of the left proximal thigh muscle. The remainder of the examination is unremarkable. X-ray of the pelvis is shown in the image ,Which of the following is most likely responsible for this patient's condition? QA Bone infection 0 B. Malignancy 0 C. Muscular dystrophy 0 D. Osteonecrosis 0 E. Slipped epiphysis 0 F. Stress fracture 0 G. Synovitis
Osteonecrosis This patient has Legg-Calve-Perthes (LCP) disease, or idiopathic osteonecrosis (avascular necrosis) of the femoral epiphysis. This condition most commonly affects boys age 3-12, with a peak incidence between age 5 and 7. Typical presentation includes chronic, progressive leg pain and/or a limp. The pain is often worse with activity and may be localized to the hip or referred to the groin, thigh, or knee. Physical examination may reveal limited range of motion, particularly internal rotation and abduction, of the hip and atrophy of the proximal thigh muscle. Although x-rays early in LCP disease may be normal, subacute or chronic symptoms usually indicate a flattened and fragmented femoral head, such as in this patient. The alternating regions of lucency and density reflect replacement of necrotic bone by new bone. Patients are made non-weight bearing and are managed conservatively with bracing or splinting Surgery may be indicated in cases in which the femoral head is not well contained within the acetabulum
A 73-year-old woman comes to the office due to dry eyes For the last year, she has had mild burning discomfort associated with a sensation of dust or sand in the eyes Recently, the patient has had difficulty reading small-print books due to decreased visual acuity She uses over-the-counter eye drops to relieve the discomfort. Her medical history is notable for hypothyroidism and osteoarthritis of the knees, which are treated with levothyroxine and acetaminophen, respectively. She drinks a glass of wine nightly and does not smoke. Vital signs are normal. On examination, the pupils are symmetrically round and reactive to light The conjunctivae are normal bilaterally, and funduscopic examination is normal. Oropharyngeal examination shows dry mucosa with mild dental caries. Neurologic examination is unremarkable. Serum chemistry panel, complete blood count, and TSH are normal. Serum antinuclear antibody assay is negative Which of the following is the most likely cause of this patient's symptoms? Q A Age-related exocrine gland atrophy Q B. Autoimmune destruction of exocrine glands Q C. Inflammatory infiltration of exocrine glands Q D. Obstruction of nasolacrimal duct Q E. Systemic sclerosis with mucosa! involvement
Q A Age-related exocrine gland atrophy
A 32-year-old African American woman is brought to the hospital due to sudden-onset left-sided weakness. According to the family, she has had fatigue, malaise, and decreased appetite over the last several weeks. She attributed these symptoms to a "viral infection" and treated herself with over-the-counter acetaminophen. Her temperature is 37.4 C (99.4 F), blood pressure is 140/86 mm Hg, and pulse is 82/min. The patient appears confused and agitated. Lungs are clear to auscultation. A 2/6 holosystolic murmur is heard at the cardiac apex Extremity examination reveals left-sided 1/5 weakness with hyperactive deep tendon reflexes. The Babinski reflex is present on the left side. Laboratory results are as follows: Hemoglobin 10.2 g/dL Platelets 64,000/mm3 Leukocytes 6'800/mm3 Blood urea nitrogen 18 mg/dL Creatinine 0.8 mg/dL Antinuclear antibody (ANA) positive Rapid plasma reagin (RPR) positive Cerebrospinal fluid cell counts are normal, and cultures are negative. Which of the following is the most likely diagnosis in this patient? Q A Antiphospholipid syndrome Q B. Multiple sclerosis Q C. Neurosyphilis Q D. Polyarteritis nodosa Q E. Rheumatic fever
Q A Antiphospholipid syndrome
A 73-year-old man is brought to the emergency department after his daughter found him on the floor. The patient says that he fell and now his left hip "hurts a lot," but he does not recall any details of the fall. Over the past several days, he has felt short of breath with a cough and intermittent palpitations that he attributed to "coming down with the flu." His other medical problems include hypertension, type 2 diabetes mellitus, chronic obstructive pulmonary disease, and benign prostatic hyperplasia His temperature is 37.3 C (99.2 F), blood pressure is 118/76 mm Hg, pulse is 116/min, and respirations are 24/min. Pulse oximetry shows 95% on 2 Umin of oxygen by nasal cannula. Breath sounds are decreased at the right lung base, and crackles are present at the left lung base. The left lower extremity is shortened and externally rotated. Initial laboratory studies, including complete blood count, glucose, and serum chemistries, are normal. The pain improves with a dose of hydromorphone. X-ray findings are shown below. Next step in management? Q A Electrocardiogram, cardiac markers, and chest x-ray Q B. Hemiarthroplasty for femoral neck fracture Q C. Intravenous crystalloids Q D. Lower-extremity skeletal traction Q E. Skilled nursing care, with hip surgery after the viral syndrome resolves
Q A Electrocardiogram, cardiac markers, and chest x-ray This patient presents after a fall with a left femoral neck fracture. Hip fractures are classified by anatomic location and fracture type into either intracapsular (eg, femoral neck and head) or extracapsular (eg, intertrochanteric, subtrochanteric). lntracapsular fractures have a higher chance of avascular necrosis; extracapsular fractures have a greater need for implant devices (eg, nails, rods) The specific surgical intervention required depends largely on the details of the individual case. In general, surgical repair should be done as soon as feasible to relieve pain, minimize complications, and reduce length of hospital stay However, surgery may be delayed up to 72 hours if needed to address unstable medical comorbidity This patient has a number of concerning features. The uncertain details surrounding his fall suggest a possible syncopal episode His tachycardia and palpitations can be due to arrhythmia (atrial fibrillation); decreased breath sounds on the right side could be due to pleural effusion; cough, dyspnea, and left basilar crackles also suggest pulmonary edema or pneumonia The patient should therefore be evaluated with chest x-ray, electrocardiogram, and cardiac markers to determine the etiology of his fall and assess preoperative risk prior to surgical correction of his fracture. If there is evidence of pulmonary edema and pleural effusion, an echocardiogram is needed. Once it is determined that he is otherwise medically stable, he may be cleared for surgical intervention (Choice 8). (Choice C) Before starting intravenous fluids, pulmonary edema (heart failure) and pleural effusion should be ruled out with chest xray. This patient is not hypotensive and likely has an intracapsular fracture, which has a relatively low risk of acute blood loss and need for intravenous fluids. (Choice D) Skeletal traction has been advocated as an interim measure for patients with hip fracture who require delayed surgical repair However, recent studies have found that traction does not relieve pain or improve surgical outcomes. (Choice E) Surgery may reasonably be delayed up to 72 hours to allow for stabilization of acute medical problems, but longer delays increase the risk of medical complications and prolong the return to premorbid functional status.
A 44-year-old woman is evaluated for generalized weakness, low-grade fever, and joint pains Her daily activities are limited due to joint stiffness, especially in the morning She has bilateral swelling of her hand joints, primarily the metacarpophalangeal joints The tip of the spleen is palpable on deep inspiration Her hematocrit is 34%. Liver and renal function tests are normal. The patient is diagnosed with rheumatoid arthritis and started on initial therapy Two months later, her joint symptoms have improved, but she returns to the physician with painful oral ulcers. laboratory results are as follows: Hematocrit 33% Aspartate aminotransferase 120 U/L Alanine aminotransferase 90 U/L Alkaline phosphatase 90 U/L Bilirubin 1.1 mg/dl Creatinine 0.8 mg/dl Blood urea nitrogen 16 mg/dl Erythrocyte sedimentation rate is 22 mm/hr. Which of the following is the most likely cause of this patient's current complaints? Q A Anticytokine agent therapy Q B. Antimetabolite agent therapy Q C. Corticosteroid treatment 0 D. Felty syndrome 0 E. Lymphoma 0 F ViraI hepatitis
Q B. Antimetabolite agent therapy Methotrexate (MTX) is the initial disease-modifying antirheumatic drug of choice in patients with rheumatoid arthritis. Common side effects of MTX include gastrointestinal symptoms, oral ulcers or stomatitis, rash, alopecia, hepatotoxicity, pulmonary toxicity, and bone marrow suppression Folic acid supplementation has been shown to reduce the incidence of adverse effects of MTX therapy without loss of efficacy
A 7-year-old boy is brought to the emergency department after falling during recess. His teacher saw him jump off a swing and fall onto his outstretched left arm. The patient developed immediate pain in the arm and now refuses to move it at the elbow. On examination, the left arm is grossly deformed, swollen, and slightly pale The radial pulse is intact. The patient's x-ray is shown below. Two hours later, while the patient is awaiting surgery, pain and swelling continue to increase despite administration of nonsteroidal anti-inflammatory drugs and opioid analgesics Which of the following complications is most likely present in this patient? Q A Avascular necrosis of the humeral physis Q B. Compartment syndrome of the forearm Q C. Injury to the brachia! artery Q D. Neurapraxia of the median nerve Q E. Venous thrombosis of the proximal upper extremity
Q B. Compartment syndrome of the forearm ompartment syndrome is a rare complication of supracondylar humerus fractures, particularly those that are displaced or occur in conjunction with forearm fractures. Initial symptoms may include increasing swelling and pain that is unresponsive to escalating analgesics
A 24-year-old woman comes to the physician complaining of knee pain. The pain is located over the anterior knee and has gradually worsened over the last 3 months. The pain is sharp and seems worse when she climbs or descends stairs. A trial of over-the-counter ibuprofen did not lessen the pain The patient's past medical history is unremarkable. She smokes a half pack of cigarettes daily, drinks socially on weekends, and lives alone in an urban apartment. Her temperature is 36.7 C (98 F), blood pressure is 110/60 mm Hg, and pulse is 68/min. Her body mass index is 26 kg/m2. Inspection of the knee is normal. Which of the following is the most appropriate next step to confirm the diagnosis in this patient? Q A Aspiration of joint fluid to examine for crystals Q B. Extension of the knee while compressing the patella Q C. Magnetic resonance imaging of the knee Q D. Palpate for tenderness at the tibial tubercle Q E. X-ray of the knee
Q B. Extension of the knee while compressing the patella Patellofemoral syndrome is a very common cause of anterior knee pain Symptoms are chronic and associated with activities such as climbing stairs with extension at the knee. Women are affected more often than men. Diagnosis is based on history and examination findings, and treatment involves stretching and strengthening the thigh muscles in addition to avoiding activities that worsen the pain.
A 21-year-old man is evaluated in the postoperative recovery unit due to worsening shortness of breath. The patient was brought to the emergency department 24 hours ago after an all-terrain vehicle rolled on top of him. He has had no significant medical problems and takes no medications. The patient frequently binge drinks alcohol and was drinking just prior to the incident. He was found to have a right femoral midshaft fracture and right foot metatarsal fractures. The patient underwent intramedullary nailing of the femoral fracture and has received intravenous fluids and blood transfusion. Cast immobilization of the metatarsal fractures was also performed Temperature is 37.7 C (99 9 F), blood pressure is 130/80 mm Hg1, pulse is 120/min, and respirations are 24/min. Pulse oximetry shows 84% on room air. The patient is restless, agitated, and confused. He has scattered petechiae on the trunk, but there are no chest wall ecchymoses Breath sounds are normal and equal in both lungs. Cardiac auscultation is normal with the exception of tachycardia Abdomen is soft and nondistended. Lower extremity examination shows postoperative changes Urgent chest x-ray reveals no pneumothorax and clear lung fields. Which of the following is the most likely cause of this patient's current condition? Q A Blood transfusion-related pulmonary capillary damage Q B. Fat globules entering the bloodstream Q C. Postoperative partial lung collapse Q D. Pulmonary edema due to myocardial dysfunction Q E. Sudden decrease in the blood alcohol level Q F Thromboembolic occlusion of the pulmonary arteries Q G. Traumatic contusion of the pulmonary parenchyma
Q B. Fat globules entering the bloodstream
A 34-year-old man comes to the office due to low back pain and tightness for the past 3 months. The pain is dull with an aching quality. It is worse at night and in the morning but improves gradually during the day as he moves around. The patient has no recent injury to account for the pain and says that it has gradually gotten worse. He has no significant medical history He lives with his wife and does not use tobacco, alcohol, or illicit drugs. Temperature is 36.1 C (97 F}, blood pressure is 120/76 mm Hg, pulse is 80/min, and respirations are 14/min. There is no tenderness over the spine or lower extremity weakness. He has a negative straight-leg test Which of the following most likely accounts for this patient's symptoms? Q A Abnormal bone mineralization Q B. Inflammation at ligamentous insertions Q C. Ligamentous sprain Q D. Lumbar disk degeneration Q E. Nerve root demyelinization
Q B. Inflammation at ligamentous insertions
52-year-old woman complains of difficulty gripping her cup of coffee in the morning. She likes to solve crossword puzzles but is unable to hold a pen properly due to stiffness in her right hand. Her symptoms gradually improve during the day and she is "fully functional" by late afternoon. Examination shows non tendernodules near the elbows over the back of her forearm. The erythrocyte sedimentation rate is 45 mm/hr. Which of the following would most likely be involved during the course of this patient's disease? QA Cervical spine 0 B. Lumbar spine 0 C. Sacral spine 0 D. Sacroiliac joints 0 E. Thoracic spine
QA Cervical spine in RA Cervical joint involvement can lead to spine subluxation ➔ spinal cord compression
58-year-old woman comes to the clinic due to a 6-month history of left knee pain that is worse with activity. She initially had pain only with ambulation but has progressively developed rest pain at night. In addition, the patient has had intermittent morning stiffness of 10-15 minutes duration. There is no associated fever or weight loss. Medical history is significant for hypertension, type 2 diabetes mellitus, gastroesophageal reflux disease, and obstructive sleep apnea Current medications include hydrochlorothiazide, lisinopril, omeprazole, atorvastatin, insulin glargine, metformin, and aspirin Temperature is 37.2 C (98 9 F) and blood pressure is 146/86 mm Hg Examination shows a mild effusion, tenderness, and decreased range of motion of the left knee. A fluctuant swelling along the posterior aspect of the knee is palpable Arthrocentesis with synovial fluid analysis reveals the following: White blood cells 1100/mm3 Gram stain no organisms Crystals none Plain film x-ray of this patient's knee joint would most likely reveal which of the following? Q A Calcification of cartilaginous structures Q B. Narrowing of joint space and osteophytes Q C. Normal joint space with soft-tissue swelling Q D. Periarticular osteopenia and joint margin erosions Q E. Punched-out erosions with a rim of cortical bone
Q B. Narrowing of joint space and osteophytes Physical examination in osteoarthritis shows crepitus, periarticular bony enlargement, and painful or decreased range of motion. Xray findings include a narrowed joint space, osteophytes, and subchondral sclerosis/cysts Patients can also have a bland synovial effusion (no organisms, <2,000 white blood cells/mm3)
A 16-year-old boy was brought to the emergency department because of left shoulder and left hand pain after falling on his outstretched hand while playing soccer. He heard a crunching sound and had intense pain in his left shoulder area following the injury. Examination shows bruising around the clavicle area. He is holding his left arm with his right hand. There is a palpable gap in the middle of the clavicle. Auscultation shows a loud bruit just beneath the clavicle. An x-ray film of the left shoulder and chest shows the middle of the clavicle is fractured and displaced. Which of the following is the most appropriate next step in management? Q A CT chest for pneumothorax Q B. Nerve conduction studies Q C. Angiogram Q D. Open reduction of the clavicle Q E. Closed reduction with figure of eight brace
Q C. Angiogram The clavicle is one of the most commonly injured bones in the body The majority of clavicular fractures occur in the middle third of the bone. Injury to this bone classically occurs during athletic events and follows a fall on an outstretched arm or a direct blow to the shoulder. Patients with clavicular fractures present with pain and immobility of the affected arm. The contralateral hand is classically used to support the weight of the affected arm. The shoulder on the affected side is displaced inferiorly and posteriorly. A careful neurovascular exam should accompany all fractures to the clavicle due to its proximity to the subclavian artery and brachia! plexus In this case, a bruit is heard and an angiogram is necessary to rule out injury to the underlying vessel The clavicle is one of the most commonly injured bones in the body The majority of clavicular fractures occur in the middle third of the bone. Injury to this bone classically occurs during athletic events and follows a fall on an outstretched arm or a direct blow to the shoulder. Patients with clavicular fractures present with pain and immobility of the affected arm. The contralateral hand is classically used to support the weight of the affected arm. The shoulder on the affected side is displaced inferiorly and posteriorly. A careful neurovascular exam should accompany all fractures to the clavicle due to its proximity to the subclavian artery and brachia! plexus Int his case, a bruit is heard and an angiogram is necessary to rule out injury to the underlying vessel
A 35-year-old man comes to the office for follow-up of low back pain. He has aching pain in the left lumbar paraspinal area that is worse at the end of the day and relieved overnight with rest. The pain began 3 months ago without any precipitating trauma. The patient was initially treated with intermittent doses of acetaminophen and naproxen but continues to have moderate residual pain. There is no associated fever, weight loss, radicular pain, lower extremity weakness, or urinary symptoms. His medical history is unremarkable. The patient is employed as a factory line worker, which requires him to lift 5-7 kg (11-15 lb) several times daily On physical examination, vital signs are normal and the patient appears comfortable. Cervical and thoracic spine range of motion is normal, but flexion and extension of the lumbar spine elicits pain Straight leg raising test is normal. There is mild tenderness in the left lumbar paraspinal tissues but no midline tenderness. Upper and lower extremity strength and deep tendon reflexes are normal. Which of the following is the best recommendation for management of this patient's pain? Q A Discontinue working until the pain is resolved Q B. Epidural glucocorticoid injection Q C. Exercise therapy Q D. Lumbar support brace Q E. Spinal manipulation therapy
Q C. Exercise therapy Management of chronic back pain should include an exercise program emphasizing stretching and strengthening of the back muscles and aerobic conditioning. Acetaminophen or nonsteroidal anti-inflammatory drugs can be used intermittently. Some patients may benefit from tricyclic antidepressants or duloxetine, but opioids, benzodiazepines, and muscle relaxants are not advised.
A 65-year-old man comes to the clinic with pain in his right shoulder over the past few weeks. He works as a house painter and has been experiencing pain when he reaches for objects or lifts his arm above his head. The patient has no recent history of trauma to the shoulder. Vital signs are normal. There is no tenderness distal to the lateral edge of the acromion or over the acromioclavicular joint Strength, sensation, and arterial pulses in the upper extremities are normal and symmetric. Passive range of motion in the shoulder is full in all directions. However, when the shoulder is internally rotated, the thumb is pointed toward the floor, and the arm is passively raised in front of the patient, the patient experiences reproduction of his pain at 60 degrees of flexion; continued passive motion above the patient's head further increases the pain. Which of the following is most likely responsible for this patient's condition? Q A Adhesions and fibrosis of the shoulder joint synovial lining Q B. Arthritic degeneration of the acromioclavicular joint Q C. Complete tear of a rotator cuff tendon Q D. Inflammation of the rotator cuff tendons Q E. Nerve root impingement at the level of the cervical spine Q F. Rupture of the long head of the biceps tendon
Q D. Inflammation of the rotator cuff tendons This patient with subacute shoulder pain on abduction most likely has rotator cuff tendinopathy (RCT) RCT results from repetitive activity above shoulder height (eg, painting ceilings) and is common in middle-aged and older individuals. Chronic tensile loading and compression by surrounding structures can cause microtears, fibrosis, aind inflammatory calcification in the rotator cuff tendons (especially supraspinatus). Pain may also emanate from the subacromial bursa and the tendon of the long head of the biceps On flexion or abduction of the humerus, the space between the humeral head and acromion is reduced, causing pressure on the supraspinatus tendon and subacromial bursa. Impingement syndrome, a characteristic of RCT, refers to compression of these soft tissue structures. Impingement can be demonstrated with the Neer test With the patient's shoulder internally rotated and forearm pronated, the examiner stabilizes the scapula and flexes the humerus. Reproduction of the pain is considered a positive test Untreated, chronic RCT can increase the risk for rotator cuff tear. Patients with a tear typically present with weakness of abduction following a fall or other minor trauma (Choice C)
A 21-year-old man comes to the office requesting a refill of an opioid medication for new-onset left hip pain The pain started 3 weeks ago and was initially only with weightbearing, but has progressively worsened and is now present at rest and overnight He has no history of trauma. Medical history is notable for sickle cell disease with several hospitalizations for acute pain crisis. His last hospitalization was 3 months ago. The patient has been taking some leftover opioid pain medications from that hospitalization, in addition to regularly scheduled folic acid and hydroxyurea He is sexually active with a new female partner The patient does not use alcohol, tobacco, or illicit drugs His temperature is 37.2 C (99 F), blood pressure is 100/70 mm Hg, pulse is 80/min, and respirations are 16/min. Physical examination reveals no local tenderness, but there is restriction of abduction and internal rotation of the left hip. The right hip and other joints are normal. Hip x-rays and erythrocyte sedimentation rate are normal. Which of the following is the most likely diagnosis? Q A Bacterial infection of the proximal femur Q B. Gonococcal infection of the synovial fluid Q C. Opioid drug-seeking behavior Q D. Osteonecrosis of the proximal femur Q E. Slippage of the femoral head epiphyseal plate
Q D. Osteonecrosis of the proximal femur Avascular necrosis Etiology steroid use • Alcohol abuse • Systemic lupus erythematosus • Antiphospholipid syndrome • Hemoglobinopathie•s (eg, sickle cell) • Infections (eg, osteomyelitis, HIV) • Renal transplantation • Decompression sickness Clinical manifestations Groin pain on weight bearing • Pain on hip abduction & internal rotation • No erythema, swelling, or point tenderness Laboratory findings Normal white blood cell count • Normal ESR & CRP Radiologic imaging Crescent sign seen in advanced stage • MRI is most sensitive modality (Choice A) Osteomyelitis is a recognized complication of sickle cell disease, and is usually due to Staphylococcus aureus or Salmonella. It is most common in children, often multifocal, and typically accompanied by fever, malaise, and elevated ESR. (Choice B) Neisseria gonorrhoeae can cause acute purulent arthritis (usually without associated skin lesions or fever). However, involvement of the distal large joints (eg, knees, wrists, ankles) is more common, and it would usually be associated with elevated ESR. (Choice C) Features that suggest drug-seeking behavior include "lost" or "stolen" medication, premature refill requests, and pain inconsistent with examination findings or known pathology. This patient's pain is easily explainable by osteonecrosis, which is a recognized complication of his underlying disease. (Choice E) Slipped capital femoral epiphysis (SCFE) is characterized by pain and altered gait. Obesity is an important risk factor. SCFE most often occurs in early adolescence and is usually visible on x-rays
A 42-year-old woman comes to the office with a 4-month history of heartburn. She describes a periodic "sticking sensation" in her chest during meals. In addition, the patient has recently been unable to participate in her normal exercise routine due to dyspnea on exertion and joint pain in her hands and feet She does not use tobacco, alcohol, or illicit drugs Lung examination reveals bilateral end-inspiratory crackles. Endoscopic evaluation shows mild hyperemia in the distal esophagus Esophageal manometry shows lack of peristaltic waves in the lower two-thirds of the esophagus and a significant decrease in lower esophageal sphincter tone. Which of the following is the most likely mechanism responsible for this patient's manometric findings? Q A Dysfunction of inhibitory neurons Q B. Eosinophilic infiltration of esophageal mucosa Q C. Loss of intramural neurons Q D. Smooth muscle atrophy and fibrosis Q E. Striated muscle inflammation
Q D. Smooth muscle atrophy and fibrosis Systemic sclerosis
A 70-year-old man comes to the physician for pain and stiffness of his neck, shoulders, and hips for the last 3 months. His stiffness is worse in the morning and lasts 1-2 hours. He also complains of general malaise and a recent weight loss of 3.17 kg (7 lb) The patient has no headache, scalp tenderness, visual symptoms, or jaw claudication. Examination shows no overt synovitis at the joints,with normal passive range of motion. The arteries of the scalp, neck, and extremities are normal on palpation without tenderness. Laboratory results are as follows Leukocytes 7500/µL Hematocrit 31% Platelets 450,000/ µL Erythrocyte sedimentation 85 mm/h Thyroid-stimulating hormone 1.8 µU/mL Creatine kinase 33 U/L Which of the following is the most appropriate next step in management of th1is patient? Q A Measure antinuclear antibodies and rheumatoid factor levels Q B. Recommend low-impact aerobic exercise Q C. Temporal artery biopsy Q D. Treatment with low-dose prednisone Q E. Treatment w.ith nonsteroidal anti-inflammatory drug
Q D. Treatment with low-dose prednisone Polymyalgia rheumatica {PMR) affects patients age >50 and is characterized by pain and stiffness in the neck, shoulders, and pelvic girdle, along with an elevated erythrocyte sedimentation rate. The treatment of choice for uncomplicated PMR is low-dose prednisone, which results in rapid relief of symptoms
A 23-year-old woman comes to the clinic due to a 3-month history of intermittent hand stiffness and pain She has also felt excessively tired and has had occasional episodes of knee pain and sharp chest pain. The patient easily sunburns with minimal sun exposure, which she attributes to the adverse effects of her acne medication. She has a history of acne vulgaris, which has been well controlled by oral minocycline The patient does not use tobacco, alcohol, or illicit drugs. Temperature is 37.2 C (98.9 F), blood pressure is 130/70 mm Hg, and pulse is 80/min. Bilateral hand joints and wrists are mildly tender and swollen. Lung fields are clear to auscultation and heart sounds are normal. The abdomen is soft and nontender with no organomegaly There is mild bilateral lower extremity edema. The patient has no skin rash or lymphadenopathy. Laboratory results are as follows Hemoglobin 12.2 g/dL Platelets 98,000/mm3 Leukocytes 3,300/mm3 Serum creatinine 0.8 mg/dl Urinalysis 2+ protein, 5-10 erythrocytes/hpf Which of the following is the best next step in evaluating this patient? Q A Anti-cyclic citrullinated peptide antibod Q B. Anti-double-stranded DNA antibody Q C. Anti-histone antibody Q D. Anti-topoisomerase antibody Q E. Antinuclear antibody
Q E. Antinuclear antibody
A 61-year-old man comes to the office with a 3-week history of low back pain that has a dull, aching quality and no radiation. It is not affected by movement but is worse at night and often wakes him from sleep The patient has no history of trauma or previous back problems. He otherwise feels well but has experienced a 5-kg (11-lb) weight loss over the last 6 months. There is no associated fever, bowel or bladder incontinence, or lower extremity weakness. Medical history is notable for hypertension, for which the patient takes chlorthalidone and amlodipine On examination, temperature is 36.7 C (98.1 F), blood pressure is 145/88 mm Hg, pulse is 76/min, and respirations are 12/min. Head and neck, cardiac, lung, and abdominal examinations show no abnormalities. Rectal examination shows normal sphincter tone and a smooth, mildly enlarged prostate There are no palpable deformities in the lumbar spine and no tenderness along the midline or in the paraspinal muscles. Lower extremity motor strength and reflexes are normal and symmetric Straight leg raise test is negative Which of the following is the most appropriate next step in management of this patient? QA Bone scan Q B. Evaluation for surgery Q C. MRI of the lumbar spine Q D. Physical therapy for supervised exercise program Q E. Plain film x-rays Q F. Trial of nonsteroidal anti-inflammatory drugs without further testing
Q E. Plain film x-rays Indications for imaging in low back pain X-ray Osteoporosis/compression fracture • Suspected malignancy • Ankylosing spondylitis (eg, insidious onset, nocturnal pain, better with movement} MRI Sensory/motor deficits • Cauda equina syndrome (eg, urine retention, saddle anesthesia) • Suspected epidural abscess/infection (eg, fever, intravenous drug abuse, concurrent infection, hemodialysis)
A 61-year-old man comes to the office due to chronic left knee pain. The pain is worse when he first gets out of bed in the morning and is associated with mild stiffness. The patient is an avid gardener, but it is becoming harder for him to kneel due to pain. He has a similar pain in his hips that is gradually getting worse. He does not take any medications and has never sought treatment for his knee pain The patient is a retired army paratrooper with no chronic medical conditions and no history of trauma. On examination, the left knee has a small effusion and mild pain over the medial and lateral joint lines. There is no clicking or locking of the knee, but crepitus is present with range of motion. The patient is able to bear weight without pain and the joint is stable to varus and valgus stress. Xray of the knee is shown in the image below Which of the following is the most appropriate initial treatment for this patient's knee pain? Q A Aspiration of the knee joint Q B. Corticosteroid injection of the knee Q C. Knee replacement surgery Q D. Placement of a pillow between the knees at night Q E. Quadriceps strengthening exercises
Q E. Quadriceps strengthening exercises This patient has chronic joint pain and stiffness consistent with osteoarthritis. The x-ray reveals typical findings, including narrowing of the joint space (most notable in the medial compartment) and periarticular osteophyte formation. Risk factors for osteoarthritis include obesity, prior joint injury, and abnormal joint alignment Initial management of osteoarthritis of the knee includes lifestyle measures such as weight loss and regular moderate activity (eg, walking) In addition, a dedicated exercise program (often with assistance from a physical therapist) to strengthen the quadriceps muscles can improve function and protect the articular cartilage from further stress. Quadriceps strength declines with age, and the pain of osteoarthritis may induce patients to decrease activity, leading to disuse atrophy of the quadriceps complex. This can cause abnormal loading of the articular cartilage and accelerate degeneration of the joint Simple analgesics (eg, nonsteroidal anti-inflammatory drugs) may also be helpful
A 49-year-old woman comes to the office due to swelling and stiffness of the hands for the past several months. For the last year, she has also had intermittent episodes of pain and bluish discoloration of the fingers, especially when exposed to cold. The patient is unable to play tennis on weekends like she did before due to fatigue and breathlessness. She takes pantoprazole for heartburn and lisinopril for hypertension The patient drinks alcohol on social occasions but does not smoke cigarettes. Temperature is 37.1 C (98 8F), blood pressure is 140/90 mm Hg, and pulse is 78/min. Physical examination shows puffiness of the hands and skin thickening in the fingers, as shown in the image below Q A Anticardiolipin antibodies Q B. Anti-cyclic citrullinated peptide antibodies Q C. Antimitochondrial antibodies Q D. Anti-neutrophil cytoplasmic antibodies Q E. Anti-smooth muscle antibodies Q F Anti-topoisomerase I antibodies
Q F Anti-topoisomerase I antibodies Systemic sclerosis
A 22-year-old man involved in a motor vehicle collision undergoes a prolonged surgery to repair a left tibial fracture and popliteal artery injury The day after surgery he has increasing pain in the left leg and is treated with intravenous fluids and morphine Two hours later, the patient continues to have pain that worsens with passive leg movement and has a "pins and needles" sensation. Examination shows tense, tender swelling of the left calf. There is sensory loss between the great and second left toes. The lowerextremity pulses are palpable bilaterally. Which of the following is the most important next step in management of this patient? Q A Elevate the leg and apply ice packs Q B. Initiate patient-controlled analgesia Q C. Obtain a Doppler ultrasound of the lower extremity Q D. Obtain x-rays of the lower extremity Q E. Return to the operating room for fasciotomy
Q E. Return to the operating room for fasciotomy Clinical features of compartment syndrome Common Pain out of proportion to injury • Pain i on passive stretch • Rapidly increasing & tense swelling • Paresthesia (early) Uncommon • ! Sensation • Motor weakness (within hours) • Paralysis (late) • ! Distal pulses (uncommon) Deep venous thrombosis (DVT) is diagnosed with Doppler ultrasonography. Like CS, DVT may also present with increasing pain and swelling, but symptoms are more insidious in onset and less severe. The presence of neurologic symptoms makes CS more likely, especially in a patient who is predisposed
A 52-year-old man comes to the emergency department with a 24-hour history of pain and swelling of his right knee. He has no fevers, chills, or recent trauma to the knee. In addition, the patient has had constipation, excessive urination, and fatigue for the past several months. His medical history is notable for a kidney stone a year ago that passed with intravenous and oral hydration The patient does not take any prescription medications, and does not use tobacco, alcohol, or illicit drugs. His temperature is 37.2 C (98.9 F) and blood pressure is 130/76 mm Hg Examination shows tenderness, erythema, and swelling of the right knee. The remainder of the examination is unremarkable. Laboratory results are as follows: Arthrocentesis is performed. Which of the following is most likely to be found on synovial fluid assessment in this patient? Q A Clear, transparent fluid Q B. Hemorrhagic effusion Q C. Needle-shaped crystals Q D. Polymorphs filled with gram-positive cocci Q E. Rhomboid-shaped crystals
Q E. Rhomboid-shaped crystals This patient has symptomatic hypercalcemia (fatigue, constipation, nephrolithiasis [likely calcium stones]), suggesting primary hyperparathyroidism He now has acute inflammatory arthritis. Pseudogout is caused by calcium pyrophosphate dihydrate (CPPD) crystals in the joint space and is a common complication of hyperparathyroidism with chronic hypercalcemia. Pseudogout is also associated with hypothyroidism and hemochromatosi
A 13-year-old boy is brought to the physician for evaluation of a limp He has had mild left hip pain for several weeks. After the boy fell off his bicycle today, his mother noticed that he was limping and brought him to the office. His body mass index is 31 kg/m2• He walks with a limp to the examination table, favoring his right leg As he sits down, his left leg is rotated externally. There is decreased range of motion and pain with internal rotation of his left hip. There is no leg length discrepancy. Bilateral anteroposterior and frog-leg lateral x-rays of both hips are obtained. The frog-leg lateral view is shown below What is the most likely diagnosis? Q A Avascular necrosis of the femoral head Q B. Chronic developmental hip dysplasia Q C. Femoral neck stress fracture Q D. Legg-Calve-Perthes disease Q E. Slipped capital femoral epiphysis
Q E. Slipped capital femoral epiphysis Slipped capital femoral epiphysis is a common hip disorder seen in overweiglht adolescents. Urgent surgical fixation is required to prevent avascular necrosis of the hip Choice A) AVN of the femoral head can be a complication of SCFE as the displaced femoral head can disrupt the blood supply (Choice B) Developmental hip dysplasia is caused by abnormal development of the hip in utero. It is usually detected on the newborn physical examination but can be diagnosed at a later age when a limp is noted. Leg length discrepancy is present and radiographs show a poorly formed femoral head, making this diagnosis unlikely (Choice C) Femoral neck stress fractures occur most commonly in runners or other athletes doing extensive training. Patients typically present with a gradual increase in hip pain, especially with activity They also have pain with passive range of motion of the hip, especially internal and external rotation. (Choice D) Legg-Calve-Perthes disease is a syndrome of idiopathic AVN of the hip that most commonly affects boys age 5-7 years
A 12-year-old boy is brought to the physician because of right groin pain, knee pain, and limping. He has had these symptoms for the past 2 weeks. He is at the 90th percentile for weight and 60th percentile for height He is afebrile, and his other vital signs are within normal limits. Examination shows that the range of motion of the right knee joint is within normal limits but hip movements are restricted and the right foot points outward. There is external rotation of the right thigh on flexion of the hip After confirming the diagnosis, which of the following is the most appropriate management? Q A Aspiration and microscopic examination of the hip joint synovial fluid Q B. Closed reduction of the hip joint Q C. Conservative management with rest and analgesics Q D. Immediate osteotomy of the femoral neck Q E. Surgical pinning of the femoral head
Q E. Surgical pinning of the femoral head Slipped capital femoral epiphysis typically occurs in obese, early-adolescent boys It should be promptly treated with surgical pinning of the slipped epiphysis where it lies (i e , in situ} in order to lessen the risks of avascular necrosis of the femoral head and chondrolysis
A 35-year-old man comes to the office with a 5-day history of back pain, which began at the end of a long day of carrying heavy boxes while moving to a new apartment Since then, he has had sharp pain in the lumbar spine radiating to the posterior aspect of the left calf and foot The patient has no associated bladder or bowel incontinence. Medical history is notable only for recurrent poison ivy dermatitis. He smokes marijuana frequently on weekends but does not use tobacco, alcohol, or other illicit drugs. On examination, vital signs are normal. Pain and temperature perception are preserved in the affected extremity as well as in the perinea! area. Anal reflex is normal. Lifting the left leg to 70 degrees with the knee held straight causes burning pain radiating from the low back to the left foot. Which of the following is the best next step in management of this patient? QA MRI of the lumbar spine Q B. Nerve conduction studies Q C. Supervised exercise program Q D. Surgical decompression Q E. Trial of nonsteroidal anti-inflammatory drugs Q F. X-ray of the lumbar spine
Q E. Trial of nonsteroidal anti-inflammatory drugs This patient has acute lumbosacral radiculopathy (sciatica}, most likely due to nerve root compression by a herniated disc. Older patients or those with prior traumatic injury can also have nerve root compression due to lumbar spondylosis. Less common causes include infectious or inflammatory disorders, mass lesions, vascular disorders, and developmental anomalies. Patients with uncomplicated lumbar strain can have radiation of pain to the buttocks or posterior thigh, but radiation to the calf and foot is more consistent with sciatica. Traction on the nerve root during the straight leg raise test causes worsening or reproduction of pain. Most patients with acute sciatica will experience spontaneous resolution; therefore, initial management is primarily focused on acute relief of symptoms. Nonsteroidal anti-inflammatory drugs and acetaminophen are the preferred first-line drugs. Short-term use of opioids or muscle relaxants can be considered in patients with persistent pain but is associated with significant sedation. Activity modification is often advisable, but patients should be encouraged to maintain moderate physical activity MRI can confirm disc herniation or other common causes of sciatica but would not change initial management Indications for MRI include progressive sensory or motor deficits, signs of cauda equina syndrome (eg, saddle anesthesia}, or concern for epidural abscess (eg, fever, intravenous drug abuse). (Choice B) Electrodiagnostic studies are indicated for patients with persistent neurologic deficits, especially those being considered or surgery or who have had previous spinal surgery. (Choice C) Physical therapy referral for a supervised exercise program can lbe considered for patients with persistent pain, but effectiveness is variable; most patients with acute back pain will experience spontaneous improvement (Choice D) Surgical intervention in sciatica is indicated for patients with rapidly progressive neurologic deficits or severe, persistent pain and disability who have not responded to more conservative measures This young patient has acute sciatica with a clear inciting event (lifting heavy objects) and no risk factors for osteoporosis (eg, corticosteroid use, heavy alcohol use) or malignancy (eg, older age, unexplained weight loss, anorexia) An x-ray would not change initial management
A 5-year-old girl is brought to the clinic with joint pain and rash. Last week, the patient developed pain in her knees. The pain resolved after a few days, but now her ankles and wrists are tender. She has also developed a nonpruritic, pink rash on her back. The patient had previously been healthy except for a sore throat a few weeks ago that resolved on its own. Temperature is 38.3 C (101 F), pulse is 85/min, and respirations are 20/min. Cardiac examination is normal. The lungs are clear to auscultation. The wrists and ankles are stiff and tender to manipulation. Multiple large, well-demarcated, erythematous, nonpruritic patches with a slightly raised outline are present on the trunk and proximal limbs. Laboratory results are as follows: Leukocytes 6,500/mm3 Neutrophils 56% Lymphocytes 33% Hemoglobin 12.5 g/dL Platelets 380,000/mm3 C-reactfve protein 32 mg/L (normal: <3 mg/L) Erythrocyte sedimentation rate 62 mm/h Which of the following is the most likely diagnosis in this patient? QA Acute rheumatic fever 0 B. Henoch-Schonlein purpura 0 C. Lyme disease 0 D. Systemic juvenile idiopathic arthritis 0 E. Systemic lupus erythematosus
QA Acute rheumatic fever
A 53-year-old man comes to the emergency department at 6:00 AM due to severe pain in his right great toe for the past 3 hours. His pain began suddenly as a dull ache, and rapidly worsened to severe throbbing that is not relieved by acetaminophen. Medical history is notable for hypertension, type 2 diabetes mellitus, and hypercholesterolemia. The patient has smoked 2 packs of cigarettes daily for 30 years He drinks 2-3 beers and 3-4 cups of coffee daily, consumes fast food often, and does not exercise regularly. Current medications include metformin, losartan, amlodipine, sitagliptin, and atorvastatin. The patient's temperature is 36.8 C (98.2 F), blood pressure is 160/90 mm Hg, pulse is 88/min, and respirations are 16/min. On examination, the right great toe appears markedly swollen, red, and warm to the touch. Which of the following interventions would be most appropriate to prevent development of further similar episodes in this patient? QA Alcohol cessation 0 B. Discontinue atorvastatin 0 C. Discontinue losartan 0 D. Low-dose prednisone 0 E. Moderation of coffee intake 0 F Smoking cessation
QA Alcohol cessation
A 48-year-old man comes to the office due to pain in his right hand. He has had symptoms intermittently for the last 5 years, but they have become worse over the last 3 months. The pain radiates to the anterior aspect of the forearm. His medical history is notable for type 2 diabetes mellitus, and his most recent hemoglobin A 1 c is 8.5%. The patient does not use tobacco, alcohol, or illicit drugs He works as an assistant manager at a supermarket Blood pressure is 148/95 mm Hg, pulse is 76/min, and respirations are 12/min. BMI is 32 kg/m2 On examination, the patient is mildly obese but otherwise appears comfortable. There is weakness of thumb opposition and slightly decreased light touch sensation over the palmar surface of the first 3 digits Which of the following is the most likely cause of this patient's hand symptoms? QA Anatomic compression of nerve 0 B. Glycosylation of proteins in vasa nervorum 0 C. Motor neuron degeneration 0 D. Neuronal destruction in sensory ganglia 0 E. T cell-mediated immune response 0 F Vasoconstriction of arterioles
QA Anatomic compression of nerve carpal tunnel syndrome is caused by compression of the median nerve as it 1Passes deep to the flexor retinaculum in the wrist Risk factors include obesity, diabetes, hypothyroidism, and pregnancy Symptoms include pain and numbness in a median nerve distribution, with thenar atrophy in severe cases
A 10-year-old girl is brought to the clinic with pain in multiple joints. Her pain began 3 months ago in her hands and has since progressed to involve her wrists, knees, and ankles. The patient previously danced competitively for the ballet team at school but has been unable to participate for the past month due to pain Her parents have been giving her acetaminophen without improvement She has had no fevers, rashes, abdominal pain, or weight loss. The patient had a febrile seizure at age 4 but otherwise is healthy. Her maternal aunt has rheumatoid arthritis, and her paternal grandmother had type 1 diabetes mellitus. Physical examination shows a comfortable young girl who grimaces due to pain when asked to walk or use lher hands. The hands, wrists, ankles, and knees are symmetrically swollen with mild erythema and warmth. Which of the following laboratory findings would most likely be seen in this patient? QA Anemia 0 B. Antibodies to double-stranded DNA 0 C. Antibodies to streptolysin 0 0 D. Hypoferritinemia 0 E. Hypogammaglobulinemia 0 F Leukopenia 0 G. Thrombocytopenia
QA Anemia Polyarticular juvenile idiopathic arthritis is an autoimmune disorder of childhood marked by symmetric arthritis of the upper and lower extremities. Laboratory findings reflect systemic inflammation, including elevated inflammatory markers and acute phase reactants as well as anemia
A 49-year-old woman comes to the office due to difficulty swallowing solid food. The patient's symptoms are worse with breads and crackers, but she has no difficulty with liquids, and she uses sips of water to help with swallowing She thinks that she may have lost weight The patient's medical history is significant for hypertension and hypothyroidism treated with amlodipine and levothyroxine, respectively She also uses over-the-counter saline eye drops for eye dryness. The patient has never used tobacco or alcohol.Family history is negative for cancer. She has been monogamous with her husband for 15 years Blood pressure is 140/90 mm Hg and pulse is 60/min. BMI is 19 kg/m2 Physical examination shows oral thrush, prominent dental caries, and bilateral, firm submandibular nodules. There is no lymphadenopathy Cardiopulmonary examination shows no abnormalities. Abdomen is soft and nontender with no organomegaly. There is no peripheral edema. Which of the following is the best next step to confirm this patient's diagnosis? QA Antibodies to Ro/SSA and La/SSB 0 B. CT scan of the neck 0 C. Eating disorder assessment 0 D. Esophageal endoscopy with biopsy 0 E. Submandibular gland biopsy 0 F Videofluoroscopic swallowing study
QA Antibodies to Ro/SSA and La/SSB Sjogren syndrome
A 35-year-old woman comes to the office due to oral ulcers. She has multiple painful sores on her oral mucosa that began a few days ago. The patient had similar lesions 3 months ago that healed without scarring. In addition, she recently was evaluated by an ophthalmologist for blurred vision and was diagnosed with anterior uveitis. She has also had recurrent genital lesions requiring frequent gynecology appointments over the last year. On examination, there are visible oral ulcerations as well as scattered hyperpigmented skin lesions and tender, indurated areas on her legs. Which of the following is the most likely diagnosis? QA Beh<;et syndrome 0 B. Herpes simplex infection 0 C. Reactive arthritis 0 D. Sarcoidosis 0 E. Systemic lupus erythematosus
QA Beh<;et syndrome
A 35-year-old woman comes to the clinic complaining of aching pain and stiffness over her entire body for the past 3 months. She also has easy fatigability, poor sleep, and frequent headaches. The patient has had difficulty performing her daily tasks and feels that her work performance as an accountant has worsened. She has used over-the-counter pain medications with no relief. On examination, she has tenderness to gentle palpation over the muscles of her neck, shoulders, and back. Vital signs are within normal limits. laboratory results show normal electrolytes, complete blood count, erythrocyte sedimentation rate, and thyroid function tests. Which of the following is the best initial therapy to improve this patient's symptoms? QA Exercise program with aerobic conditioning 0 B. High-dose prednisone 0 C. Hydroxychloroquine 0 D. Low-dose prednisone 0 E. Methotrexate 0 F Oxycodone/acetaminophen
QA Exercise program with aerobic conditioning This patient's presentation is consistent with likely fibromyalgia (FM) FM presents most commonly in young to middle -aged women with widespread pain, fatigue, and cognitive/mood disturbances. Patients tend to have a fairly normal physical examination except for point muscle tenderness in areas such as the mid trapezius, lateral epicondyle, costochondral junction in the chest, and greater trochanter. FM has no specific diagnostic laboratory findings Revised 2010 American College of Rheumatology criteria suggest using the widespread pain index and symptom severity scale, rather than trigger points, for diagnosis. The index and scale better emphasize cognitive problems, fatigue, and severity of somatic symptoms. Initial management of FM involves a multidisciplinary approach including patient education about FM, regular aerobic exercise, and good sleep hygiene Education requires validating that FM is a benign condition with a favorable prognosis Although patients may complain of increased short-term pain during or after activity, a regular exercise program (eg, aerobic conditioning, strength training, stretching) improves long-term pain. Water exercises also can greatly reduce the pain Medications (eg, tricyclic antidepressants) are reserved for patients failing initial measures. There is limited evidence for other approaches, including trigger-point injections, analgesics, and alternative therapy (eg, tai chi, yoga)
A 51-year-old woman comes to the clinic due to 6 months of fatigue and low back pain radiating to the buttocks. She also has persistent muscle pain in her arms and shoulders that worsens acutely after exercise. Physical examination shows normal muscle strength. Her joints are not swollen, but palpation over the outer upper quadrants of the buttocks, mid trapezius, and medial aspect of the knees elicits tenderness. C-reactive protein level is 3 mg/L (reference value <8 mg/L). Which of the following is the most likely diagnosis? QA Fibromyalgia 0 B. Giant cell arteritis 0 C. Polyarteritis nodosa 0 D. Polymyalgia rheumatica 0 E. Polymyositis 0 F Rheumatoid arthritis 0 G. Seronegative spondyloarthropathy
QA Fibromyalgia
A 6-year-old boy is brought to the office for evaluation of leg pain. The pain has been constant for the last 2 weeks and keeps the patient from playing with his friends. It is worse at night and has made it difficult for the child to fall asleep. He wakes up several times nightly to void and has had episodes of incontinence as the leg pain prevents him from walking to the bathroom. The patient has had no recent fever or dysuria. Temperature is 36.7 C (98 F). Cardiac examination reveals tachycardia; no murmurs are present A soft, non-fluctuant, tender, 5-cm (2 5-in) mass over the right anterior distal thigh without overlying erythema is noted. The right knee has full range of motion, and no effusion is noted. An erythematous, papular rash is present over the chest, trunk, and groin. Laboratory results are as follows Serum chemistry Sodium150 mEq/L Potassium3.8 mEq/L Skeletal survey reveals a single small, well-defined, lytic distal femoral diaphyseal lesion. Which of the following is the most likely diagnosis in this patient? QA Langerhans cell histiocytosis 0 B. Multiple myeloma 0 C. Osteogenesis imperfecta 0 D. Osteoid osteoma QE Osteomyelitis nF OsteosaFcoma
QA Langerhans cell histiocytosis This child has a lytic bone lesion, rash, and signs of diabetes insipidus (ie, polyuria, hypernatremia) consistent with Langerhans cell histiocytosis (LCH) LCH is a rare disorder in which histiocytes (macrophages within tissue) proliferate and infiltrate one or more organ systems Bone is most commonly affected, but invasion of the skin (eczematous rash), lymph nodes, lungs (cough, pulmonary nodules), liver, spleen, and CNS (central diabetes insipidus with polyuria and hypernatremia) can also occur. Bone lesions are frequently found in the skull; the femur, vertebrae, and other bones can also be affected. Lesions may be asymptomatic or associated with local pain and an overlying tender mass. X-ray shows characteristic "punched-out" lytic lesions; biopsy is required to confirm the diagnosis. Although the bone tumors are benign, they can be locally destructive and are typically treated with curettage or chemotherapy (eg, prednisone) Osteogenesis imperfecta presents with blue sclerae, diffuse cortical thinning, attenuation of trabeculae, and pathologic fractures. A solitary lytic bone lesion would not be expected (Choice D) Osteoid osteoma is a benign bone tumor that typically presents in adolescents with bone pain that is worse at night and resolves with nonsteroidal anti-inflammatory medications. A small, round hypodensity is seen on x-ray Osteoid osteoma is less likely in this child with concomitant rash and diabetes insipidus (Choice E) Osteomyelitis generally presents acutely with bony tenderness, swelling, and fever. Although initial x-rays may be normal, later they may show cortical loss and new bone formation. This patient has no fever, and osteomyelitis is not associated with rash and hypernatremia. (Choice F) Osteosarcoma is the most common primary bone malignancy and presents with localized pain and swelling in long bone metaphyses X-ray findings include periosteal elevation (Cadman triangle} and a "sunburst" pattern of new bone formation, which are not seen on this patient's x-ray
A 23-year-old woman comes to the office due to right foot pain that started 6 weeks ago, is localized to the forefoot, and has worsened over the past week. The patient reports no trauma or inciting event, and the pain is preventing her from doing her usual exercise. She takes no medications. She runs 3 times a week and occasionally goes hiking. The patient does not use tobacco, alcohol, or illicit drugs. Her last menstrual period was 3 weeks ago, and she is not sexually active. Temperature is 37 C (98 6 F}, blood pressure is 100/60 mm Hg, pulse is 70/min, and respirations are 12/min. Her BMI is 22 kg/m2 Physical examination of the right foot shows a clicking sensation when the third and fourth metatarsal heads are squeezed together; this also reproduces the pain in the plantar surface. The rest of the physical examination shows no abnormalities. Whic'h of the following is the most likely diagnosis? QA Morton neuroma 0 8. Plantar fasciitis 0 C. Stress fracture 0 D. Tarsal tunnel syndrome 0 E. Tendon injury
QA Morton neuroma Morton neuroma is a mechanically induced degenerative neuropathy that causes numbness, aching, and burning in the distal forefoot from the metatarsal heads to the third and fourth toes. Treatment is conservative, with metatarsal support or padded shoe inserts Plantar fasciitis causes focal pain in the plantar area of the rear foot It usu·ally worsens with the first steps in the morning, decreases with activity during the day, and often worsens again later in the day with prolonged weight bearing Examination will show point tenderness at the plantar surface of the heel. Stress fractures occur due to a sudden increase in exertion without adequate rest that eventually breaks the bone. Patients have sharp, localized pain and tenderness over a bony surface. The female athlete triad (oligomenorrhea, decreased caloric intake, and osteopenia/osteoporosis) is associated with stress fracture, but this patient has normal weight and no reported menstrual irregularities Tarsal tunnel syndrome is due to compression of the tibial nerve as it passes through the ankle and is usually caused by a fracture of the bones around the ankle. Patients have burning, numbness, and aching of the distal plantar surface of the foot or toes that sometimes radiate up to the calf Tendinopathies are common in runners and can involve the ante.rior tibial, posterior tibial, peroneal, Achilles, or flexor hallucis longus tendons. Patients develop localized pain and swelling in the ankle and hindfoot Forefoot tendinopathies are less common
A 60-year-old man comes to the physician complaining of fatigue and muscle weakness in his extremities. He has also lost 7 kg (15 lb) during the last 3 months. He has no other medical conditions. The patient smokes 2 packs of cigarettes daily and consumes alcohol occasionally. His muscle strength is 3/5 in the proximal muscle groups symmetrically. His reflexes are 2+ bilaterally. No sensory abnormality is present There are erythematous to violaceous papules involving the dorsum of his fingers. Chest x-ray reveals an ill-defined mass in the right lower lobe. His muscle weakness is most likely due to a lesion involving which of the following? QA Muscle fibers 0 B. Peripheral nerves 0 C. Postsynaptic membrane 0 D. Presynaptic membrane 0 E. Spinal cord
QA Muscle fibers Dermatomyositis is an idiopathic inflammatory myopathy with immune-mediated muscle injury that can be due to a paraneoplastic syndrome in malignancy Patients typically have symmetrical proximal muscle weakness and erythematous rash over the dorsum of the fingers (Gottron's sign) and/or upper eyelids (heliotrope eruption)
A 13-year-old boy is brought to the office due to progressive pain of the left knee for 3 months. The pain initially occurred only after basketball practice but improved with rest and ibuprofen. However, over the last several weeks, the patient has had knee pain within the first 15 minutes of practice. Last week, the pain became so severe that h:e missed practice and a game. The patient has had no fever, weight loss, or night sweats. The pain has not caused him to awaken. Vital signs are normal. Physical examination reveals tenderness over the left tibial tubercle, which appears more prominent than the right The left knee has full range of motion and no palpable effusion. Extension of the left knee against resistance and squatting reproduces the pain Which of the following is the most likely cause of this patient's knee pain? QA Osgood-Schlatter disease 0 B. Patellar tendinitis 0 C. Patellofemoral pain 0 D. Prepatellar bursitis 0 E. Tibial osteomyelitis
QA Osgood-Schlatter disease Osgood-Schlatter disease, a common cause of knee pain in young adolescents, is caused by a traction apophysitis of the tibial tubercle. Patients typically have pain exacerbated by activity, and examination reveals prominence and tenderness over the tibial tubercle. Patellar tendinitis is an overuse injury due to repetitive jumping (eg, basketball, volleyball) Patients have anterior knee pain that worsens with activity Unlike Osgood-Schlatter disease (in which pain occurs over the tibial tuberosity), in patellar tendinitis, point tenderness is present at the inferior pole of the patella. (Choice C) Patellofemoral pain can be an overuse injury presenting as progressive knee pain in the setting of running and squatting Pain is usually localized to the patella and associated with a sensation of instability or "buckling" at the knee. (Choice D) Prepatellar bursitis occurs after direct, chronic trauma (eg, wrestiling) of the anterior knee. Symptoms include pain with direct pressure and superficial swelling over the patella, not the tibial tuberosity (Choice E) Tibial osteomyelitis may present with local (eg, pain, erythema) and systemic (eg, fever) inflammation, in addition to refusal to bear weight on the affected extremity Pain does not improve with rest alone.
A 30-year-old man comes to the office due to shortness of breath, nonproductive cough, and fatigue for the past 3-4 weeks. He also has been urinating more than usual. The patient has no other medical issues and currently takes no medications. He has smoked a pack of cigarettes daily for 8 years. Vital signs are normal. Cardiopulmonary examination shows scattered crackles but is otherwise unremarkable. The abdomen is soft and nontender. Laboratory results are as follows Complete blood count Hemoglobin 12 g/dl Platelets 200,000/mm3 Leukocytes 7500/mm3 Serum chemistry Sodium 140 Potassium 4.2 Urea nitrogen 20 Creatinine 0,8 Calcium 11 Glucose 80 Chest x-ray reveals bilateral hilar fullness and interstitial infiltrates. Results of HIV testing and purified protein derivative skin testing are negative. Which of the following is the best therapy for this patient? QA Antifungals 0 B. Antituberculosis agents 0 C. Bisphosphonates 0 D. Chemotherapy QE Glucocorticoids QF Parathyroidectomy
QE Glucocorticoids sarcoidosis
A 54-year-old man comes to the physician complaining of morning facial puffiness and bilateral leg swelling His other medical problems include recurrent pulmonary infections due to bronchiectasis and psoriasis. His blood pressure is 143/92 mm Hg and pulse is 92/min. His body mass index is 24 kg/m2. Examination shows a fourth heart sound. Hepatomegaly, palpable kidneys, and 2+ pitting edema of the lower extremities to the knees bilaterally are present on examination. Urinalysis shows 4+ proteinuria and normal urinary sediment Which of the following is the most likely diagnosis in this patient? QA Amyloidosis 0 B. Hepatorenal syndrome 0 C. Hypertensive nephrosclerosis 0 D. lgA nephropathy 0 E. Polycystic renal disease
a- amyloidosis this patient's clinical presentation of facial swelling, bilateral lower-extremity edema, and massive proteinuria are consistent with nephrotic syndrome The presence of nephrotic syndrome with palpable kidrneys, hepatomegaly, and ventricular hypertrophy (suggested by an audible fourth heart sound) in the setting of chronic inflammatory disease (recurrent pulmonary infections, bronchiectasis) is suggestive of secondary amyloidosis (AA) as a common unifying diagnosis Treatment is usually directed at the underlying inflammatory disease. Colchicine is approved for both the treatment and prophylaxis of AA.
A 15-year-old boy comes to the physician with left shoulder joint pain and swelling. He has had these symptoms for the past 3 months and they are getting progressively worse. He has no fever, weight loss, or night sweats. The patient lifts weights and plays baseball 5 days a week. He has taken acetaminophen and ibuprofen with little relief. Examination shows a tender mass at the left proximal humerus. Range of motion is nor.ma!. Laboratory evaluation shows elevated! alkaline phosphatase, erythrocyte sedimentation rate, and lactate dehydrogenase X-ray of the shoulder is shown in the photograph below Which of the following is the most likely diagnosis? QA Ewing sarcoma 0 B. Osteoid osteoma 0 C. Osteomyelitis 0 D. Osteosarcoma 0 E. Stress fractur
osteosarcoma the patient most likely has osteosarcoma, the most common primary bone tumor affecting children and young adults. Boys between ages 13 and 16 years are at higher risk. In children, the tumor occurs most friequently at the metaphyses of long bones such as the distal femur, proximal tibia, and proximal humerus. Constitutional symptoms such as fever, weight loss, and malaise are usually absent On physical examination, the most important finding is a tender soft-tissue mass. Characteristic x-ray findings include a spiculated "sunburst" pattern (yellow arrow) and periosteal elevation known as the Codman triangle (red arrow) Alkaline phosphatase and lactate dehydrogenase are elevated from turnover of damaged osteocytes; high levels may correlate with adverse prognosis. Increased erythrocyte sedimentation rate is a non-specific marker of inflammation. Treatment includes tumor excision and chemotherapy
A 68-year-old woman comes to the office due to a 1-year history of right leg pain. The pain has a relatively constant severity and has not responded to over-the-counter medications. The patient has also experienced intermittent headaches over the last 6 months. Medical history is notable for hypertension, diet-controlled diabetes mellitus, and acute cholecystitis for which she underwent cholecystectomy 5 years ago Her current medications include amlodipine and as-needed acetaminophen The patient does not smoke, and she drinks 1 or 2 alcoholic beverages a night. Vital signs are normal. The hip and knee have normal range of motion with no gross deformity X-ray of the skull reveals areas of bone resorption and sclerosis. X-ray of the leg reveals cortical thickening with mild bowing Which of the following are the most likely findings on laboratory evaluation? serum ca serum phosphate Alkaline phosphatase urine hydroxyproline
serum ca normal serum phosphate normal Alkaline phosphatase high urine hydroxyproline high Pagets disease of bone